Probability Notes
Probability Notes
net
C~APT.E~ ,,-OUR
Theory oJ Probability
- 4·1. Introduction. I( an e'xperimept is ,repeated under essentially
homo.geneous and similar conditio.ns we generally co.me, across two. types o.f
situatio.ns:
(i) The result o.r what is usually kno.wn as-the 'O.utccUI'Ie' is UIliijue o.r certain.
(ii) The result is no.t unique but may be o.ne o.f the several possible o.utco.mes.
The pheno.mena cov~re4 by.Jj) are kno.wn as 'deterministic' or 'predictable'
pheno.mena By a deterministic pheno.meno.n we m.ean:o.n~ ill whi~1\ ltle result, can
be predicted with certainty. Fo.r example:
(a) Fo.r a_~d~t g~,
V oc: p1 ,
I.e., PV = constant,
provided the temperature temairi~ the·same.
(b) The velocity 'v' o.f a:particle after. time 't ' is given by
v =u +at
where u is the initial velocity and a is the acc~leratio.n. Thisequatio.n uniquely
determines v if the.light-hand quantitie~)81'e kno.wn.
,- La"
(c)Oh ms . C'£
w, VIZ., = ii
where C is the flow o.f current, E-the potential difference.between the two. ends.o.f
the conductor and R the resistance, uniquely determines the vallie C as soon as E
and R are'given:
A deterministic model is defined as a model which stipulates' that the co.ndi-
tions under which an experiment is perfo.rmed determine the o.utco.me o.f the
experiment Fo.r a number o.f situatio.ns the deterministic modeitsuffices. Ho.wevef,
there are pheno.mena [as covered by (U) above] -which do. no.t leml themselves to
deterministic approach and are kno.wn as 'unpred,ctable' or 'probabilistic'
pheno.mena. Fo.r ex~,ple l '
(i) In tossing o.f aco.in o.ne is not sUre if a head o.r tail wiD- be obtained.
(Ii) If a light ture has' lasted for I ho.urs, no.thing can be said atiout its further
life. It may fail to. function any mo.ment
In such cases we talk o.f chanceo.r pro.babilitY which is't8J(en to'be il quantitative
measure o.f certaioty.
4·2. Short· History. Galileo (1564-1642), an Italian m'athematic~, was the
first to attempt at a quantitative'measure o.fpro.bability while dealing with some
problems related to the theory o.f dice in gambiing. Bul the flfSt fo.undatio.n o.f the
mathematical theory ifprobabilily was laid in the mid-seventeenth century by lwo.
French mathematicians, B. Pascal (1623-1662) andP. Fermat (1601-1665), while.
Visit : www.EasyEngineering.net
Visit : www.EasyEngineering.net
Visit : www.EasyEngineering.net
Visit : www.EasyEngineering.net
(i) In tossing an·unbiased or unifonn com, head or tail ate ~uatly likely
events. •
(a) In throwing an unbiased die, all the six faces are equally likely to'come.
Independent events. Several ev.ents are said 'to be indepeildtnt if the
happening (or non-happening) of an 'event is 'not affected by the ~..w.lementary
knowledge concerning the occurrence of any n\llllbet: of the remaining events. For
example
. (i) In tossing ..an un1;>iased cpin the event of.g~tin,g Jl head in the flrSt toss
is independent of getting a head,in the s~nd, third and subsequent throws.
(ii) ~f we draw a card from ,a ~lc of well-shuffled cards apd.replace it
before dr!lwing. the secQnd card, th~ trsult of the seco~d draw is indepcfnden~ of
the fIrSt draw. But, however, if the first card draWn is not replaced then the second
draw is dependent on the first draw. . •
..·3·1. Mathemat,ical or Classical or ~a·priori~ rrobabalitY
Definition. If a ttial results in n exhaustive, mutually exclusive and equally
likely cases an<J m 9f them are favourable to the happening of an event E. ·then -the
probability 'p' of happening of E is.given by
I .. it'! J
Visit : www.EasyEngineering.net
Visit : www.EasyEngineering.net
4·4 Fundamentals of Mathematical Statistics
Visit : www.EasyEngineering.net J
Visit : www.EasyEngineering.net
Theory of Probability 4-5
Example 4,2. A bag contains 3 red, 6 white and 7 blue balls, What is the'
probability that two balls drawn are white and blue?
Solution. Total number of balls = 3 + 6 + 7= 16,
Now, out of 16 balls, 2 can be drawn in ItlC2 ways,
, 16 16 x 15
, . Exhausuve number of cases = C2= 2 - 120,
Out of 6 while balls 1 ball can be drawn in tiCI ways and out of 7 blue balls 1
ball can be drawn in 7CI ways. Since each of the fonner cases can be assoc~:ed
with each of the latler cases, total number of favourable cases is: tlCI x 7Ct
=6x7= 42,
' ed probab'li
ReqUlr 42 = 20'
I ty= 120 7
Example 4,3. (a) Two cards are drawn 'at random from a well-shuJPed pact
of52 cards, Show that lhe chance of drawing two aces is 11221,
(b) From a {kick of52 cards, three are drawn at random, Find the chance
that they are a king, a queen and a knave,
(c) Four cards are u·3wnfrom a pack of cards, Find tMprobability that
(i) all are diamond, (ii) there is one card of each suit, and (iii) there are
two spades and two hearts,
Solution. (a) From a pack of 52 cards 2 cards can be drawn -in S2C2 ways,
all being equally likely,
, , Exhaustive number of ·cases =. 52C2
In a pack there are 4 aces'and therefore 2 aces can be drawn iit
' ed
ReqUlr b b'l' ·C2 4 x 3 2
" pro a Iity = S2C2 = -2- x 52 x 51
(b) Exhaustive number of cases .= 52C, I
A pack of cards contains 4 kings, 4 9\leens and 4 knaves. A king, a queen and
a knave can each be drawn in ·CI ways and since each way of drawing a king can
be associated with each of the w~ys of drawing a queen and Ii knave, the total
numberoffavourablecases=·CI x·CI x ·CI
R 'ed bab'l' _ ·CI x-·G1'X·CI 4 x4 x.4 X6_~
,, equlli pro J Ity - 51C, =52 x 51 x 50 - 5525
(c) Exhaustive number of cases = SlC. •
(i) Required probability = 13C.
SlC.
13C C 13 13,.. 13C
(ii) ReqUJ'red probab'l' ~x I X .,..1 X L
Ilty=---""'"'------::S::---'---
• 2C.
13CZ X 13C2
(iii) Required probability = --:::---
52C•.
Visit : www.EasyEngineering.net
Visit : www.EasyEngineering.net
Example 4·4. What is the probability of getting 9 cards of the same suit in
one hand at a game of bridge?
Solution. One hand in a game of bridge consists of 13 cards.
:. Exhaustive number of cases = SlC13
Number of ways in which, in one hand, a particular player gets 9 cards of one
suit are 13C, and the number of ways in which the remaining 4 cards are of some
other suit are 39C4. Since there are 4 suits in a pack of cards, total num\:ler of
favourable cases = 4 x nC, x 39C4.
4x nC x 39C
Required probability = ' ..
SlCn
Example 4·5. (a) Among the digits 1,2,3,4,5, atfirst one is chosen and then
a secon,d selection is made among the remaining foUT digits. Assuming t!wt all
twenty possible outcomes have equal probabilities,find the probability'tllDt an odd
digit will be selected (i) tIlL. first time, (ii) the second time, and (iii) both times.
(b) From 25 tickets, marked with the first 25- numerals, one is drawn at
random. Find the chance that
(i) it is a multiple of5 or 7,
(ii) it is a multiple of3 or 7. •
Solution. (a) Total number of cases = 5 x 4 = 20
(i) Now there are 12 cases in which the first digit drawn is·O(Id, viz., (1,2).
(1,3), (1,4), (1,5), (3, 1), (3, 2), (3,4), (3, 5), (5, 1), (5, 2), (5, 3) and (5,4).
:. The probability that the flTSt digit drawn is odd
12 3
= 20= '5
(ii) Also there are 12 cases in which the second digit dtawn is odd" viz .•
(2, 1), (3, 1), (4, 1), (5,1), (1, ~), (2, 3), (4, 3), (5, 3), (1, 5), (2, 5), (3,5) and (4, 5).
. . The probability that the second digit drawn is odd
12 3
= 20 = '5
(iii) There are six cases in which both the digits drawn are odd, viz., (1,3),
(1,5), (3, 1), (3, 5), (5, 1) and (5, 3).
:. The probability that both the digits drawn are odd
6 3
=20=10
(b) (i) Numbers (out of the first 25 numerals) which are multiples Of 5 are 5,
10. 15,20 and 25, i.e., 5 ~n all and the numbers which are multiples of 7 are 7. 14
and 21, i.t., 3 in all. Hence require(f'number of favourable cases are 5+3=8.
. . Required probability = ~
(ii) Numhers ~among the first 25 numerals) which are multiples of3 are 3, 6,
9, 12, 15, 18,21,24, i.t., 8 in all, and the numbers which are multiples of 7 are 7,
Visit : www.EasyEngineering.net
Visit : www.EasyEngineering.net
14,21, i.e., 3 in all. Since the number 21 is common ill both the cases, the required
number of distinct favourable cases is 8 + 3 - 1 = 10.
.. 'd probab'l'
Requrrc 10 .= '2
1 rty = 25 5
Example 4·6. A committee of 4 people is to be appointed from J officers of
the production department, 4 officers of the purchase department, two officers of
the sales department and 1 chartered aCCOi4ntant, Find the probability offoT1ning .
the committee in the following manner:
(i) There must be one from each category.
(U)It should have at least one from the purchase department.
The chartered accountant must be in the committee.
(iii)
Sol"tion. There are 3+4+2+ I = I0 persons ~n ull and a committee of 4 people
can be fonned out of them in 10C" wa)'$. Hence exhaustive number eX cases is
10C" = 10 x 9 x 8 x 7 :.: 210
4!
(i) Favourable number of cases for the committee to consist of 4 members, one
from each category is :
"CI x 3CI X lCi X 1 = 4 x 3 x 2 = 24
. ed probab'l'
ReqUlf 24 8
I Ity = 210 = 70
Visit : www.EasyEngineering.net
Visit : www.EasyEngineering.net
(b) What is the probability that four S's come consecutively in the word
'MISSISSIPPI' ?
Solution. (a) The word 'REGULATIONS' consists of II letters. The two
letters Rand E can occupy llp2 , i.e. ,II x 10 = 110 -p0sitions.
The number of ways in which there will be exactly 4 letters between Rand E
are enumerated below:
(i) R is in the 1st place and E is in the 6th place.
(ii) R is in the 2nd place and E is in the 7th place.
...
~
(viii) S •S S S
Since in eac~ of the above cases, the total number of arrangements of the
remair.ing 7 letters, viz." MIIlPPl of which 4 -are of one kind, 2 of other kind
and cne ·Jf tJlird kind are 4! ~: I! ' the required nwnber of favourable cases
8 x 7!
= 4!
~! I!
R 'red b bT 8 x 7! II!
equt pro a 1 lty = 4! 2! I! + 4! 4! 2! I!
.. 8x7!x4! 4
= 11 ! = 165
Visit : www.EasyEngineering.net
Visit : www.EasyEngineering.net
Theory of Probability 49
l~
4 (,) 4
(il) 1 .16 4,5,6 3 x 3=9
(iil) 2
5 (,)
(li) g 5
1
20 5,6 2 x 2= 4
6 (i) 6
(il) 1
(iiI)
(iv) {i 2
3
24 5,6 4 x 2= 8
Visit : www.EasyEngineering.net
Visit : www.EasyEngineering.net
4-10 Fundamentals of Mathematiul Statistics
= P [(2t - 3n)(2x - n) :S OJ
=P [ x lies between ~ and 3;]
3n n
Favourable I3Jlge = "2 - '2 = n
TQtall3Jlge=- 2n
;=. Required probability = .!!... = !
2n 2
Example 4·10. Out of(2n+ 1) liclcels consecutively numbered thiee are drawn
at random. Find tM chance that the numbers on tMm are in A.P.
[Calicut Univ. B.Sc., 1991; Delhi Univ. B.Sc.(Stat. HODS.), 1992]
Solution. Since out of (2n + 1) tickets, 3 tickets can b€f drawn in, 2n + lC3
ways,
Exhaustivenumberofcases:2n+1C3= (2n + 1) 2n (2n - U
3 !
-_ n (4n2 -1)
- 3
To find the favourable number of cases we are to enumerate all the cases in
which the numbers on the drawn tickets are in AI' wid) common difference, (say
d= 1,2,3 •...• n-l,n).
Visit : www.EasyEngineering.net
Visit : www.EasyEngineering.net
'Theory of Probability 4·11
211-1
2, n +], 211
l
If d:= /1- I, the possible cases are as follows:
I, n,
3. II + 2, 211 + I
• i.e., 3 cases in all
EXCERCISE 4 (a)
1. «(1) Give the classical and stati"stical definitions of probability. What are
the objections raised in these definitions?
[Delhi Univ. B.Sc. (Stat. Hons.), 1988, 1985)
(b) When are a number of cases said to be equalIy likely7 Give an example
each, of the following:
(i) the equally likely cases.,
(it) four cases which are not equally likely, and
(iit) five ca~es in which one case is more likely than the other four.
(c) What is meant by mutually exclusive events? Give an example of
(I) three mutually exclusive events,
(ii) three events which are not mutualIy exclusive.
[Meerut Univ. B.Sc. (Stat.), 1987]
(d) Can
(i) events be mutually exclusive and exhaustive:?
(it) events be exhaustive and indepenene!
(iii) events be mutually exclusive and independent?
.(il') events be mutua1\y exhaustive, exclusive and independent:?
2. (a) Prove that the probability of obtaining a total of 9 in a'single throw
with two dice is one by nine.
(b) Prove that in a single throw with a pair of dice the probability of getting
the sum of7 is equal to 1/6 and the probability of getting the sum of 10 is equal
to 1112.
(c) Show that in. a single throw with two dice, the chance of throwing more
than seven is equal to that of throwing less than seveh. .
Ans.51l2 [Delhi Univ. B.Sc., 1987, 1985]
(d) In a single throw with two dice, what is the number whose probability
is minimum?
Visit : www.EasyEngineering.net
Visit : www.EasyEngineering.net
(e) Two persons A and B throw three dice (SIX faced). If A throws 14, findB's
chance of throwing a higher number. [Meerut Univ. B.Sc.(Stat.), 1987]
3. (a) A bag contains 7 white, 6 red and 5 black balls. Two balls are drawn at
random. Find the probability that they will both be white.
Ans. 21/153
(b) A bag contains 10 white, 6 red, 4 black and 7 blue balJs. 5 balls are drawn
at random. What is the probability dlat 2 of them are red and one black?
ADS. 'C2 x 4CI/ %1Cs
4. (a) From a set of raffle tickets numbered 1 to 100, three are drdwn at random.
What is the probability that all the tickets are odd-numbered?
Ans. soC, /IOOC3
(b) A numtler is chosen from each of the two sets :
(1,2,3,4,5,6,7,8,9); (4,5,6,7,8,9)
If Pl' is the probability that the sum of the two numbers be 10 and P2 the
probabili~y that their sum be 8, fmd PI + P2.
(c) Two different digits are chosen at random from the set 1.2.3,... ,8. Show
that the prol:ability that the sum of the digits will be equal to 5 is the same as the
probability that their sum will exceed 13, each being 1/14. Also show that the
chance of both digits exceeding 5 is 3/28. [Nagpur Univ. B.Sc., ~991]
5. What is the chance that (i) a leap year selected 8t random will contain S3
Sundays? (a) a non-leap year selected at random would contain 53 Sundays.
Ans. (i) 2fT, (ii) In
6. (a) What is the probability of having a knave and a queen when two cards
are drawn from a pack of 52 cards?
ADS. 8/663
(b) Seven cards are drawn at random from a pack of 52 cards. What is the
probability that 4 will be red and 3 black?
Ans. 26C4 x 211iC3 /S2C,
(c) A card is drawn from an ordinary pack and a gambler bets that it is a spade
or an ace. What are the odds against his winning the bet?
Ans. 9:4
(d) Two cards are drawn from a pack of 52 cards. What is the chance tIlat
(i) they belong to th: same suit?
(ii) they belong to different suits and different denominations.
[Bombay Univ. BoSe., J986)
7. (a) If the letters of the word RANDOM be arranged at random, what is the
chance ~t there are exactly two 1eue~ between A and O.
(b) Find the probability that in a random arrangement of the leters of the wool
'UNIVERSITY', the two I's do not come together.
Visit : www.EasyEngineering.net
Visit : www.EasyEngineering.net
Visit : www.EasyEngineering.net
Visit : www.EasyEngineering.net
13. In a box there are 4 granite stones, 5 sand stones and.6 !>ricks of identical
size and shape. Out of them 3 are chosen at random. Find the chance that :
(i) They all belong to different varieties.
(ii)They all belong to the4Same variety.
(iii)They are all granite stones. (Madras Univ. B.Se., Oct. 1992)
14. If n people are seated at a round table, what is the chance that two named
individuals will be next to each other?
Ans. 2/(n-l)
15. Four tickets marked 00, 01, 10 and 11 respectively are placed in a bag. A
ticket is drawn at random five times, being replaced each time. Find th,e probability
that the sum of the numbers on tickets thus drawn is 23.
[Delhi Univ. B.Sc.(Subs.), 1988]
16. From a group of 25 persons, what is the prob~i1ity that all 25 will have
different birth4ays? Assume a 365 day year and that all days are equally likely.
[Delhi Univ. B.5c.(Maths Hons.), 1987]
Hint. (365 x 364 x ... x 341) + (365)15
4·4. Mathematical Tools: Preliminary Notions or Sets. The set theory was
developed by the German mathematician, G. Cantor (1845-1918).
4·4·1. Sets and Elements 0( Sets. " set is a well defmed collection or
aggregate of all possible objects having giv(' 1 properties and specified according
to a well defined rule. The objects comprisin~ a set are called elements, members
or points of the set Sets are' often denoted by c''PitaI letters, viz., A, B, C, etc. If x
is 8..1 element of the set A, we write symbolically x E A (x belongs. to A). If x is not
a member of the set A, we write x E A (x does not belong to A). Sets are often
described by describing the properties possessed by ~ir members. Thus the set
A of all non-negative rational numbers with square less than 2 will be written as
A ={x: xrational,x ~ O,~ < 2}.
If every element of the set A belongs to the set B, i.e., if X.E A ~x E B, then
we say that A is a subset of B and write symbolically A ~ B (A is contained in B)
or B :2 A (B contains A ). Two sets A and B are said to be equq/ or identical if
A~ B andB~A andwewriteA=BorB=A.
A null or an empty set is one which does not contain any element at all and is
denoted by +.
Remarks. 1. Ev~ry set is a subset of it'self.
2. An empty set is subset of every set.
3. A set containing only one element is :onceptually distinct from the element
itself, but will be represented by the sarm. symbol for the sake of convenience.
4. As will be the case in all our applications of set theory, especially to
probability theory, we shall have a fIXed set S (say) given in advance, and we shall
Visit : www.EasyEngineering.net
Visit : www.EasyEngineering.net
be concerned only with subsets of this given set. The underlying set S may vary
from one application to another! and it will be referred to as universal set of each
particular discourse.
4.4·2, Operation on Sets
The union of two given sets A and B, denoted by A u B, is defined as a set
consisting of all those points which belong to either A or B or both. Thus
symbolically.
AuB={x :xeAorxeB).
Similarly
u" Ai =(x : x e Ai for at least one i = I • 2 , ... , n )
i= 1
The intersection of two sets A and B, denoted by A () B. is defined ag a set
consisting of all those elements which belong to both A and B. Thus
A ()B = (x: x e A and x e B) ..
Similarly
"
('\ Ai = (x : x e Ai for all i =1, 2, ...• n)
i= 1
For example, irA = (I. 2. 5. 8. 10) and B = (i. 4. 8. 12). thell
AuB =(l.2.4.5,8.IO.12) andA() B:i: (2.8).
If A and B have no common point. i.e., A () B ~ •• then the sets A and Bare
said to be disjoint, mutually exclusive or non-overlapping.
The relative difference of a setA from another setB, denoted by A-B is defined
as a set consisting of those elements of A which do not belong to B. Symbolically.
A-B= {x :xe Aandx~ B}.
The complement or negative of any set A, denoted by A is it set containing all
elements of.the universal setS. (say). that are not elements of A, i.e .. if == S -A.
4·4· 3, Algebra or Sets
Now we state certain imponant properties concerning operations on sets. IfA,
Band C are the subsets of a universal set S. then the following laws hold:
Commutative Law A uB=BuA.A ()B=B()A
Associative Law =
(A () B) u C A u (B Y C)
(A ()B) () C=A () (B ()C)
Distributive Law A () (B u C) = (A () B) u (A .() C)
A u (B () C) =(A u B) () l-\ u C)
Complementary Law : =
A u A S • A () A= •
Au S=S ,( ',' A =S> ,A ()S=A
Au.=A.A().=.
Difference Law A - B =A () B
A - B = A - (A () B) =(A u B) - B
A - (B-C) =(A -B) u(A - C).
Visit : www.EasyEngineering.net
Visit : www.EasyEngineering.net
(A u B) - C =(A - C) u (B - C)
A - (B u C) =(A - B) ()(A - C)
=
(A () JJ) u (A - B) A , (A () B) () (A - B) =~
De-Morgan's Law-Dualizalion Law.
(A u B) =A () B and """'(A-()-'B=) =A u B
More generally /
n n n n
( U Ai) = () Ai and ( () Ai) = u Ai
;=1 ;=1 ;=1 ;=1
Involution Law (A) = A
ldempotency Law AuA=A, A()A=A
4·4·4. Limit or Sequence or Sets
Let (A-) 'be a sequence of sets in S. The limit supremum or limit superior of
Ithe sequence , usually written as lim sup A., is the set of all those elements which
belong to A. for infmitely many n. Thus
lim sup A. ={x : x E A. fqr infmitely many n } J
n-too . ... (4·3)
The set of all those elements which belong to A. for all but a finite number of
n is called limit infinimum, or limit inferior of the sequence and is denoted by lim
inf A•. Thus
lim Inf A. = (x: x E A- for all but a finite number of n )
n-too ...(4·3 a)
The sequence {An} is said to have a limit if and only if lim sup A.
=lim inf A. and this common value gives the limit of the sequence.
GO ...
Visit : www.EasyEngineering.net
Visit : www.EasyEngineering.net
4·4·5. Classes of Sets. A group of sets wi1l be tenned as a class (of sets). Below
we shall define some useful typeS of clas~
A ring R is a non-empty class of sets which is closed under the fonnation of
'finite unions' and 'difference',
i.e., if A E R, B E R, then A u B E R and A - B E R .
Obviously ~ is a member of every ring.
A[r.eld F (or an algebra) is a non-empty class of sets which is closed under
the formation of finite unions and under complementation. Thus
(i) A E F, B E F => A u B E F and
(ii) A E f => A E F.
A a-ring C is a non-empty class of sets which is closed under the formation
of 'countable unions' and 'difference'. Thus
00
Visit : www.EasyEngineering.net
Visit : www.EasyEngineering.net
Visit : www.EasyEngineering.net
Visit : www.EasyEngineering.net
(ii) any trial results in,an outcome that corresponds to one an4 only one element
of the setS:
The set S associated with an exp~iment E, real or conceptual, satisfying the
above two properties is called the sample space of the experiment.
Remarks. 1. The sample space serves as universal set for all questions
concerned with the experiment.
2. A sample space S is said to be finite (infinite) sampJe sapce if the number
of elements in S is fmite (infinite). For example, the sample space associated with
the experiment of throwing the coin until a head appears, is infmite, with possible
sample points
{rot. CI)z, CI>J, ro4, ., •. }
where <.01 =11, CIh =TH, CI>J =TTH, <Jl4 =TITH, and so on, H denoting a head and
Ta tail.
3. A sample space is called discrete if it contains only finitely or infinitely
many points which can be arranged into a simple sequence rot. (Oz, •••, while a
sample space containing non· denumerable number. 9f points is called a continuous
sample space. In this book, we shall restrict ourselves to discrete sample spaces
only.
4·5·2. Event. Every non·empty subset A o( S, which is a disjoint union of
single el~.ment subsets of the sample space S of a random experiment E is called
an event The notion of an event rOay also be defmed as folJows:
"Of all the possible outcomes in the sample space of an experiment,some
outcomes satisfy a specified description,which we call an event."
Remarks. 1. As the empty set ~ is a su,?set of S ,. is also an event, known as
impossible event
2. An event A. in particular, can be a single element subset of S in which case
I
Visit : www.EasyEngineering.net
Visit : www.EasyEngineering.net
2. Let us consider a single toss of a die. Since there are six possible outcomes.
our sample space S is now a space of six points (0)1, (J)z, •••• 0>6) where O)j cor-
responds to the appearance of number i. Thus S= {Ct>t. 0)2..... 0>6} = ( 1,2.....6) and
n(S)=6. The event that the outcome is even is represented by the ~t of points
{00z. 004. 0>6} •
3. A coin and a die are tossed together. For this experiment. our sample s~ce
consists of twelve points {O)I, (J)z...... 0)12) where O)i (i = 1,2, ... , 6) represents a
head on coin together with appearance of ith number on the die and {))j
(i =7, 8, ... , 12) represents a tail on coin together with the appearance ofith number
on die. Thus
S = {O)" (J)z, ..., 0)12 ) = { (H • T) x ( I, 2, .... , 6 )} and n ( S ) = 12
Remark. If the coin and die are unbiased, we can see intuitively that in each
of the above examples, the outcomeS (sample points) are equally likely to occur,
4. Consider an experiment in which two balls are drawn one J>y one from an
urn containing 2 white and 4 blue balls such that w.hen the second ball is drawn,
the fust is not replaced.
Le~ us number the six balls as 1,2, 3,4, 5 and 6, numberS I and 2 representing
a white ball and numbers 3, 4, 5, and 6 representing a blue ball. Suppose in a draw
we pick up balls numbered 2 and 6. Then (2,6) is called an outcome of the
experiment It should be noted that the outcome (2,6) is (Jifferent from the outcome
(6,2) because in the former case ball No.2 is drawn fust and ball No.6 is drawn
next while in the latter case, 6th ball is·drawn fust'and the second'ball is drawn
next.
The sample space consists of thirty points as listed below:
O>t =(1,2) (J)z =(1,3) O>.J =(1,4) 014 =(1,5) ro, =(1,6)
CI>6 =(2,1) CJ>, =(2,3) roe =(2,4) ffi9 =(2,5) 0)10 =(2,6)
O)n =(3,1) 0>t2 =(3,2) 0>t3 =(3,4) 0>14 =(3,5) O)ts =(3,6)
O)ll; =(4 11) ~17 =(4,2) O>ta =(4,3) 0>19 =(4,5) 0hD =(4,6).
O>zt =(5,1) ron =(5,2) 0>zJ =(5,3) CIh4 =(5,4) roz, =(5.6)
0>76 =(6,1) ron =(6,2) (J)zs =(6,3) <lh9 =(6~4) CIJJo =(6,5)
Thus
$ = (rot. (J)z, ro" ... , ro,o) and n(S) ~ 30
~ S=(1,2.3,4,5,6}x{I.2,3,4,5.6)
- ((1, 1). (2. 2), (3, 3), (4,4), (?, 5), (6,6)l
The event
(i) the first ball drawn is white
(ii) the second ball drawn is white
(iii) both the balls drawn ar~ white
(iv) both the bans drawn are black
ace represented respectively by the following sets of points:
{O)" 0>%, 0>.3, 004, 0)5, 0>6, CJ>" CIla, (J)g, roto),
{O)" 0>6, O)n, ro12, 0)16, 0)17, Ohl, (j}zz, 0>76, 0>z7},
Visit : www.EasyEngineering.net
Visit : www.EasyEngineering.net
(0)1,0>6), and
(O>n, 0>14, O>IS, 0>18, CJ)19 0>20, C!h3, 0>1.4, 0>25, O>2a, 0>29,
h O>:lo),
5. Consider an experiment in which two dice are tossed. The sampl~ s~ce S
for this experiment is' given by
S = 0,2,3,4,5, 6}.x (1,2,3,4','5,6)
and n(S)=6x6=36,
Let EI be the event that 'Ilie sum of the spots on the dice is greater than 12',
E2 be we event that 'the sum of spots on the dice is divisible by 3', and E3 be the
event dtat-'the sum is ·greater than or equal tb two and is less than or equal i6 12'.
Then these events are represented by the following subsets of S :
EI =(ell }, E, =S and
El = ((1,2), (I, 5), (2, I), (2, 4), (3, 3), (3, 6), (4,2),
. (4, 5), (5, I), (5~ 4), (6, 3), (6,6)} ,
Thus n (E 1)=O, n (El )=12, and n (E,)=36
Here E is an 'impossible event' and E, a 'certain Ifvent~.
6, Let E denote tfte experiment of tossing a coin three times i~ succession or
toSsing three coins at.a time. Then the sample space S is given by
S =(H, T) x (H, T) x (H, T)
= (H, T) x (HH,HT, TH, TT)
=(HHH, HHT, lITH, HIT, THH, THT" TfH, 'T.IT)
= (0)1t 0>2. 00" .... roe). say.
If E I is the event that 'the number of heads exceeds the nUfQber' of tails' , £1,
E,.
the event of 'getting two heads' and the event of getting 'heaitin,the frrst trial'
then these are represented by the following .Sets of pOin~ ,:
EI =(O>~. 0>2, 00" rosL
, El = {0>2, 00" O>s}
and E, = (O>!, 0>2, 00,. 0>4).
7. In the fQl'eg,oing examples the sample sapce. is fi~ite. To construct an
experiment in Whict) the sample sapce is countably infmite 1 we toss a coin
repeatedly until head or tail apPears twice in succ~sion. The sample space of all
the possible outcomes may be represented as : .
~ ={HH, TT, THH, HTT, IITHH, THIT, THTHH; HTHIT, ...1.
4·5,4. Algebra of Events. For events A, B, C
(i) A u B= {O> E S : 0> E A or 0> E B}
(ii) A n B= (0) E S : 0> E A and 0> E B),
(iii) A (A complement) = {O> E S : 0> ~ A }
(iv) A - B = (0) E S: 0> E A but 0> IE B)
/I /I
(v) Similar generalisations for u Ai, n Ai, u Ai etc.
\ i .. l i=1 i
(vi) A c B ~ for every 0> E A. 0> E B.
Visit : www.EasyEngineering.net
Visit : www.EasyEngineering.net
Example 4· 11. A, B and Care three orbiirary eventS. Firid eXpressionsfor the
events noted below, in the context ofA, B and C.
(i) only A ~curs,
(ii) Both A andB, but not C, occur,
(iii) ~II three events occur,
(iv) At least one occurs,
(v) At least two occur,
(vi) OM and no more occurs,
(vii) Two' and no more occur,
(viii) None occurs.
Solution.
(i) A ()B () C, (il) A ()B () C, (iiI) A ()B () C,
(iv) ,A, u,B u C,
Visit : www.EasyEngineering.net
Visit : www.EasyEngineering.net
Visit : www.EasyEngineering.net
Visit : www.EasyEngineering.net
Visit : www.EasyEngineering.net
Visit : www.EasyEngineering.net
Visit : www.EasyEngineering.net
Visit : www.EasyEngineering.net
be specified, vii., the sample spaceS, the a-field (a-algebra) B fonned from certain
~subset of S and set function P. The triplet (S; B, P,) is often called the prohability
space. In most elementary applications, S is finite and the a-algebra B is taken to
be the coUection of aU subsets of S.,
3. It is interesting to see that there are some formal statements of the properties
of events dertved from the frequency approach. Since P(A)=ft/AIN, it is easy to see
thatP(A) ~ O. as in Axiom 1. NextsinceNs =N, P(S)=I. as in Axiom 2. In case
of two mutually exclusive (or disjoint) events A and B defined by sample points
NA and N•• the sample points belonging 10 A u B are NA + N•• Therefore.
P(AUB)=NA ;N. ~ + ':;=P(1)+P(B), as in axiom 3.
Extended AXiom or Addition. If an eventA can materialise in the 9Ccurrence
of anyone of the pairwise disjoint events At, A2, ..• so that
00
then
00
...
peA) =P ( U Ai) = l: peA) ...(1)
i= 1 i= I
Axiom ofCoDtinuity. H Bit B1• ••••• B•• ••• be a countable sequenCes of events
suchlhat_
(i) Bi ~ Bi+1' (i = I, 2, 3, ...)
and
-
(ii) n B.=.
11=1
i.e .• if each succeeding event implies the preceeding event and if their simul-
taneous occurrence is an impossible event then
-lim PCB,,) = 0 ...(2)
,.......
We shal~ now prove that these two axioms, viz., the extended axiom ofaddition
and axiom of continuity are equivalent, i.e., each implies the other, i.e., (1) ~ (2).
Theorem 4·1. AXiom ofcondnuityfollowsfrom lhe extended axiom ofaddit.ion
and vice versa.
Proof. (a) (1) ~(1). Let (B.) be a countable sequence of events such that
B1 ~ B1 ~ B, ;=l •••• ~.B,,::l 811+ 1 ::l •••
and let for any n ~ 1.
Visit : www.EasyEngineering.net
Visit : www.EasyEngineering.net
J--J.--J----.. B... 1
B..,.10 B'..,.1
Thus B. baS been expressed as the countable union of pairwi~ disjoint events
and hence. by the extended axiom of addition, we get
GO
= !, P(BIB'I+J),
k=1I
since, from (.)
P( (.) BI ) = P(~) = 0
k~1I
Further, from (**), since
00
L P (BIB'I+J)=P(BJ) S 1,
k=1
the right hand sum in (•• ), being the remainder after n terms of a convergent series
tends to zero as n-+oo.
Hence
00
Visit : www.EasyEngineering.net
Visit : www.EasyEngineering.net
B,,= u Ai .•.(4)
;=" , '.
Obviously B" is a decreasing sequence of e,vents. i.e.,
B1 ~Bl ~ ..• ~B.~B.. 1 ~ •••• ..•(5)
Also we have
A=(
"
U Ai) uB" l •••(6)
i= 1-
Since Ai's are pairwise disjoint, we get
Aj " , Bu 1 = ~. (i = 1. 2•...• n) •..(00)
"From'(4) we see that if'the event B. has occwyed it implies the "OCcurrence of
anyone of the ~vents A.+ 1• Al~;;•..• Without loss of generality lerus assume that
this event is Aj (i =II + 1. II + 2•..• ). Further since ~ 's are p3irwise disjoint, the
occurrence of Ai implies that events Ai+1tAi+2 •••• do riot Occur leading 10 the
conclusiQn that Bi + 1, Bi + 2t ••• will not occur.
00
•••(1)
;=Il
From (5) and (7) .. we observe that both the conditioQs of axiom' of continuity
are satisfied and hence we get
••.(8)
Visit : www.EasyEngineering.net
Visit : www.EasyEngineering.net
00
Visit : www.EasyEngineering.net
Visit : www.EasyEngineering.net
s
Hence by axiom 3, we get
A =
P (B) P (A () B) + P CA () B)
=
::) P (A () B) p. (B) - P (A () B)
Remark. Similarly, we shall get
_....-4,-B P(A()B)= P(A)- P(A()B)
1/B c A, then
Theore.... "·6.
(i) P (A () B)::: P (A) - P (B) ,
(U) P (B) ~ P (A)
Proof. (i) When B c A, B and A () B are r__--------.
mutually exclusive events and their union isA S
Therefore
P (A) = P [ B u (A () B) ] A
=P (B) + P (A () ]i) (By axiom 3]
::) P·(A () 11) = P (A) - P (B)
(ii) Using axiom I,
. p' (A () 11) ~ 0 => P (A) - P (B) ~ 0
Hence P (B) ~ P (A) or
C()r. Since (A () B) c A and (A () B) c B,
P (A () B) ~ P (A) and P (A () B) ~ P (B)
"·6·2. Law of Addition of Probabilities
Statement. If A and B are any (WO events I subsets ofsample space SJand are
not disjoint, then
P (A uB) = P (A) + P(B) - P (A () B) .••(4'5)
Proof.
S
A()B
A B
Visit : www.EasyEngineering.net
Visit : www.EasyEngineering.net
We have
AvB= Av(AnB)
Since A at! .1 (A n B) are disjoint.
P (A v B) = P (A) + P (A n B)
= P(A)+ [P(AnB)+ P(AnB)]r P(A.nB)
= P (A) ~ P [ (A n B ) u (A n B):] - P (A n B)
[.: (A nB) and (A nil) are disjoint]
=> P (A v B) = P (A) + P (B) - P (A n B)
Remark. An alternative PI09f is provided by Theorems 44 and 4·5.
4·6·3. Extention of General Law or Addition of Probabilities. For n events
Ah A2• ...• A .. we have
II II
P(v Ai) = ~ P(Ai)- IIi P(AinAj)+ ~ P(AinAjnA..')
;=1 ;=1 ISi<jSII IS~<j<kSr
- ... + ~_1)"-1 P (AI nt\2 n ... nA,.) ...(4·6)
Proof. For two events A 1 and A 2. we have
P (AI v A2) = P (AI) + P (A 2) - P (AI n Az) ...(.)
Hence (4·6) is true for n = 2.
Let us now suppose that (4·~) is true for n =r, (say). Then
r r
P ( vA;) = ~ P (Ai) - ~~P (Ai n Aj) + ... + \.- ,),-1 P (AI nA2n... riA)
i= 1 i = l I S i < j S.r •••( . .)
Now
r+l r
P( V Ai) = P[( v A;) V A.. l]
i=1 i=1
r r
= p ( v A;).+ P (A.. 1) - PH v Ad nA.. 1)]. ••• [Using (.~]
i .. l i=1
r r
= P( V Ai) + P (A.. 1) - P [ V (Ai n A.. 1) ] {Distributive Law)
i=1 i=1
r
= ~ P(Ai)- ~ P(AinAj)+ .. ,
i=1 ISi<jSr
Visit : www.EasyEngineering.net
Visit : www.EasyEngineering.net
r
- [ 1: P(AirtA,+I)- I I P(AirtAjrtA,+I)
;= I I ~;<j~r
SI = 1: pi= 1: P (Ai)
,; =I ; =I
S2= ~ pij= ~ P (AirtAj)
l~i<jSIl 1~;<j,~11
Visit : www.EasyEngineering.net
Visit : www.EasyEngineering.net
l'he probabi1i~ie~ P(AI), P,{Al)' •.., P(AN) Qf the mQtuaJ.lx ~xclusive forms of
A are known as the partial probabilities. Since p(A) is their sum, it may be called
the total probability. of A. Hence the name of the theqrem.
Theorem 4·7. (Boole's ine9uality). For n events AI. Al , .•.• , AN' we have
(a) "
P ( (i Ai) ~ " P (Ai) - (n - 1)
1: ...(4·7)
i=1 i=1
Visit : www.EasyEngineering.net
Visit : www.EasyEngineering.net
Now
r+ 1 r
P ( u Ai),= P ( u Ai U Ar+ I)
i:=1 ;=1
r
~P ( u Ai) + P (Ar+I) [Using (***)]
i= 1
r
~ .1; P (Ai) + P (A,+ I)
,= 1
r+ 1 r+ 1
.~ P ( U Ai) ~ I P (Ai)
;=1 i=1
Hence if (4·7a) is true for n=r, then it is also true for n=r+1. But we have
proved in (* ..) that (4·7a) is true for n=2. Hence by mathematical induction we
conclude that (4·7a) is tole for all positive integral values of n.
Theorem 4·8. For n events Al, A20 •.•• A..
/I II
P[ u Ad ~ ~ P (Ai) - I .P (Ai () Aj)
i=1 ;=1 ISi<jSII
[Delhi Univ. B.sc. (Stat Hans.), 1986]
Proof. W~ shall prove this theorem by the method of induction.
We know that
P (AI u Al u A3) = P (AI) + P (Al) + P (A3)
- [P(A I () A1) + P(A1 () A3) + P(A, () AI)] + P(A I· () Al () A,)
3 ~ .
=> P ( u A;) ~ 1; P (A;) - ~ P (Ai () Aj)
i=1 i=1 ISi<jS3
Thus the result is true for n=3. Let US now suppose that the result is true for
n=r (say). so that
r
P( \..J.Ai)~ I P(Ai)- ~ P(Ai()Aj) ...(*)
i='1 i=1 ISi<jSr
Now
r+ 1 ,r
p( U Ai)'=P( u AiUA,+I)
i= 1 ;.'1
r r -
= P ( U ,Ai) + P (Ar+ 1) - P [( U Ai) () A,+ t1
i=1 i=1
Visit : www.EasyEngineering.net
Visit : www.EasyEngineering.net
+ P (Art.) -
"
P[
r
u (A; n
i= 1
.
A,+.)] ...(
[From (.)J
..
)
Visit : www.EasyEngineering.net
Visit : www.EasyEngineering.net
.,.
4·36 Fundamentals of ~thematlcal Statistics
Visit : www.EasyEngineering.net
Visit : www.EasyEngineering.net
Theory of Probability
Visit : www.EasyEngineering.net
Visit : www.EasyEngineering.net
or equivalently,
P {none of the given events happens}
= 1 - P {at least one of them happens}.
Theorem 4·9. For any three events A, Band C
e
P (A uBI C) = (A I C) + P (B I C) - P (A n B I C)
Proof. We have
P(AuB)= P(A)+ P(B)- P(AnQ)
~ P[(A n C) u (B n C) ] = P (A n C) + P (B n C) - P (A n B n C)
Dividing both sides by P (C), we get
P[(A n C) u (B n C)] = P(A n C)+P(B n C) - P(A nB n.C) P(C) 0
P(C) P(C)' >
_ P(A n C) P(B n C) P(A nB nC)
- P(C) + P(C) .. ' - P(C)
~ P [(A u B ) n C] = P (A I C ) + P ( B I C ) - P ( A n B Ie)
P (C)
~ P [(A u B )J C] = P (A I C J+ P (B I C ) - P (A n B I C )
Theorem 4·10. For any three events A,B and C
P (A n B I C) + P (A n B I C) =P (A I C)
Proof. P(AnBI C)+ P(AnBI C)
_ P(AnBn C) P(AnBnC)
- P(C) + P(C)
_ P (A n Hn 9 + P (A n B n C)
- P(C)
= P(AnC)= P(AIc)
P (C)
Theorem 4·11. For a fued B with P (B) > 0, P (A 18) is a probability
IUlletion. [Delhi Univ. B.Sc. (Stat. HODS.), 1991; (Maths Hons.), 1992]
Proof.
nI P (A I B-) = P (A n B) ~ 0
P (B)
( . ') P (S I B) = P (S n Bl = P (B) = 1
II P (B) P (B)
(iii) If (All) is any finite or infinite sequences of disjoint events, then
P [( u A.) n B ] P [( u A.. B ) ]
/I /I
P [,; AlliS] = . P(B) = --P-(B-)--
L P(AIIB)
= /I
P (B)
= ~ r
P (All B)]
~ l- P (B)
=~
~
P (A I B)
•
Hence the theorem. /I /I
Visit : www.EasyEngineering.net
Visit : www.EasyEngineering.net
P{B0Gf":IA)u(BnCnA)
==
P(A)
P[Bnc'nA) + P(BnCnA) . .
== P(A) - f!(A) (Usmg aXIOm 3)
= P[(BnqA)+(BnCnA)]
Now Bc C => B n C = B
" P(CIA) ==P(B IA)+ P(BnCIA)
=> P(C 1A) ~ P(B IA)
4·7·3. indc))endent E,'cnts. An event B is said to be independent (or
statistically independent) of event A, if the conditional probabili~v of B given
A i.e., P (B 1 A) is equal to the unconditional probahiliiy of B, i.e., ({
P (B 1 A) == P (B)
Since
P (A n B) == P (B 1A) P (A) = P (A 1 B} P (B)
and since P (B 1 A) = P (8) when 8 is independent of A, we mLlst have
P (A 1 B) = P (A) or it fqIlows that A is also independent of B. Hence the
events A and B are independent jf and only if .
P (A n 8) = P (A) P (B) ... (4'9)
4·7'4. Pitil'1vise Indc,)cndcnt El'ents
Dcfinition. A set ofevents AI' .4 2' ••• , An are said to be pair-wise independent
if P(A;nAj )==P(A;)P(.9 rt i*j ...(4'10)
Visit : www.EasyEngineering.net
Visit : www.EasyEngineering.net
4·40
.
FundamentaJ~ o~
sample. space for a number of events. The events in S are said to be mutually
indepdendent if the probability of the simultaneous occurrence of (any) finite
number of them is equal to the product of their separate probabilities~
If Al , A z, ... , A. are n events, then for their mutual independence, we should
have
(i) P (Ai (') Aj) = P (Ad P (Ai), (; # j ; i., j = 1,2, ... , n)
(ii) P (Ai (') Aj(') AI) =P (Ai) P (Aj) P (AI), (i # j #k; i;j; k= 1,2, "',!l)
Visit : www.EasyEngineering.net
Visit : www.EasyEngineering.net
Visit : www.EasyEngineering.net
Visit : www.EasyEngineering.net
= P ( C) P ( A u B) = R.H.S.
Hence ( A u B ) and C are independent.
Theorem 4·16. If A, Band C are random events in a sample space and if A,
B andC are pairwise independent and A is independenfo/( B u C), thenA,B and
C are mutually independent.
Proof. We are given
P(AnB)= P(A)P(B) )
P(BnC)= P(B)P(C)
P (A n C) = P (A) P ( C) ... (.)
P[An(BuC)] = P(A)P(BuC)
Nc..... P [A n (B u C)] = P [ (A n B) v( An C) ]
= R (A nB) + P (A n C), - P [A nB )·n (A n C)]
= P (A) . P (B) + P (A ) . P ( C) - P (A n B n C) ...(.. )
and P (A)P (B u C) = P (A)[P (B) +P (C) -P (B n C)]
= P (A). P (B) + P (A) P (C) - P (A) P (B n C) ...( ••• )
From (.. ) and (••• ), on using (.), we get
P (A n B n C)=P (A)P(B n C) =P (A) P (B) P (C)
Hence A, B, C are mutually independent.
Theorem 4·17. For any two events A and B,
P (A nB )~P (A )~P (A uB )~P (A )+P(B)
[PatDa Univ. B.A.(Stat. HODS.), 1992; Delhi Univ. B.Sc.(Stat. Hons.), 1989]
Proof. We have
A=(AnB)u(AnB)
Using axiom 3, we have
P (A )·=P [(A nB) u (AnB)] =P(A nB)+P(A nB)
Now P [(A nB) ~ 0 (From axiom 1)
.. P(A)~ P(AnB) ..(.)
Similarly P (B )~ P( An B )
.....
:::) P(B)-·P(AnB)~ 0
Now P(AuB)= P(A)+[P(B)-P(AnB )] ( ) ...
•. P(AvB)~ P(A) :::) P(A)~ P(AuB) ( ) ...
Also P(AuB)~ P(A)+ P(Bt [From (.. )]
Hence from (.), (•• ) and (••• ), we get
P(AnB)SP(A) S P(AuB)~ P(A)+ P(B)
Aliter. Since An B c A, by Theorem 4·6 (ii) page 4·30, we get
P (A n B ) S P ( A ).
Also A c (A u B) :::) -p (A) S P (A u B)
P (AuB )=P(A )+P (B )-P (AnB)
SP(A)+P(B) [.: P (A nB) ~O]
Combining the above results,y.'e get
P(AnB)SP(A) S P(AuB)S P(A)+ P(B)
Visit : www.EasyEngineering.net
Visit : www.EasyEngineering.net
Example 4·12. Two dice, one green and the other red, are thrown. Lei A be
the event that the sum of the points on the faces shown is odd, and B be the event
of at least one ace (number' I').
(a) Describe the (i) complete sample space, (ii) events A, B, Ii, An B,
Au B, and A n Ii and find their probabitities assuming that all the 36 sample
points have equal probabilities.
(b) Find the probabilities of the events:
(I) (A u Ii) (ii) (A n B) (iii) (A n B) (iv) (A n B) (v) (A n Ii) (vi) (A u B)
(Vil)(A U B)(viil) A n (A u B )(it) Au (it n B )(x)( A / B) and (B / A ), and
(xi)( A / Ii) and ( Ii -/ A) ..
Solution •• (a) The sample space consists of the 36 elementary events .
(I,I) ;(1,2);(1,3) ;(1,4);(1,5) ;(1,6)
(2, 1) ; (2, 2) ; (2, 3) ; (2, 4) ; (2, 5 ) ; (2, 6 ) '-
(3, 1) ; (3,2) ; (3, 3) ; (3,4) ; (3, 5 ) ; (3,6)
(4,1) ;(4,2);(4,3) ;(4,4);(4,5);(4,6)
( 5, 1) ; (5, 2) ; (.5, 3) ; (5,4) ; (5, 5 ) ; (5, 6 )
(6, 1) ; (6, 2) ; ( 6, 3) ; (6, 4) ; (6, 5 ) ; (6, 6 )
where, for example, the 'ordered pair (4, 5) refers to the elementary event that the
green die shows 4 and and the r~ die shows 5.
A = The event that the sum of the numbers shown by the two dice is odd.
= ( ( 1,2); (2, 1 ) ; (1,4) ; (2,3) ; (3,2); (4,1 ) ; ( 1,6); (2,5)
(3,4) ; ( 4,3) ; (5,2) ; (6, 1 ) ; ( 3,6); (4,5); (5,4); (6,3)
( 5, 6 ) ; ( 6 , 5)} and therefore
P (A) = n (A) = ~
n (Sj 36
B = The event that at least one face is 1,
= ( ( 1, 1) ; (1,2) ; (1,3) ; (1,4 ) ; (1,5) ; (1,6)
( 2, 1) ; (3, 1) ; (4, 1) ; (5, ,1) ; (6, 1 ) ) and therefore
P (B) = !!.@. =
n(S)
.!!
36
B = The event that each of the face obtained is not an ace.
={(2,2); (2,3); (2,4); (2,5 -); (2,6); (g, 2 ); (3,3);
(3,4) ; (3,5) ; (3,6); (4,2) ; (4,3); (4,4) ; ( 4,5) ;
(4,6); (5,2)·; (5,3); (5,4); .("5,5)'; (5,6); (6,2) ;
( 6, 3) ; (6,4) ; (6,5); (6,6) -f and therefore
P (Ii) = n (Ii) = 25
n (S) 36
A·n B = Ttte ~vent that sum is odd and at least one face is an ace.
={( 1,2) ; (2, 1) ; ( 1,4) ; (4', 1) ; (I, 6) ; (·6, 1 )}
. • P (A n B) = n (: (~)B) :6 = i
Visit : www.EasyEngineering.net
Visit : www.EasyEngineering.net
A () B ={( 1,2) ; (2; 1) ; (1,4) ; (-2,3) ; (3,2) ;.( 4,1) ; ( 1,6) ; (2,5)
( 3,4) ; (4,3) ; (5,2) ; (6, 1 ); (3; 6); (4,5); (5,,4); (6,3 )
( 5,6) ; (6,5); (1,1) ; (1,3) ; (1, 5 ); (3, 1) ; (5,1 ) }
• P ~A B) = n (A u B) == 23
.. \ U n (S) 36
A ("I B ={( 2, 3 ) ; ( 3, 2 ) ; ( 2, 5 ) ; ( 3,4) ; ( 3, 6 ) ; ( 4, 3 ) ; ( 4, 5 ) ; ( 5, 2)
( 5,4) ; (5,6); (6,3); (6,5) J
P(A("IB)= n(A("IB)=Q=.!
n (S) 36 3
(b) (i) P (A u B) = P (};riB) = 1 - P (A ("I B) = 1 - !=~
6 6
, 23 13
(ii) P (A ("I B) = P~) = 1 - P (A u B) = 1- - = -
36 36
- 18 6 12 I
(iii) P(A("IB)= P(A)- P(A("IB)=-- -= -=-
3636363
(iv) V'l n B) = P (B) - P (A
P IA ("I B) = -11 - -6 = -S
36 36 36
-r=-n
(v) = 1 - P (A B) = 1 - -6I = -6S
P (It ("I D) ("I
= (1 - 36 .!! .!!)+
- 2- = ~
36363
(vii) P ~) = 1 - P (A u B) = '1 _ 23 = Q
36 36
(viii) P,[A ("I (A u B)] =P,[(A ("lit) u (it B)] ("I
= P(AnB)= 2-
36
(a)P [A u (A ("I B) ] = P (A) + P (A ("I B) - P (Ii ("I A ("I B)
18 S 2J
= P (A) + P ( "TIt ("I B) =36 +, 36 = 36
Visit : www.EasyEngineering.net
Visit : www.EasyEngineering.net
Visit : www.EasyEngineering.net
Visit : www.EasyEngineering.net
Visit : www.EasyEngineering.net
Visit : www.EasyEngineering.net
Example 4·17. Why does it pay to bet consistently on seeing 6 at least once in
4 throws ofa die, but not on seeing a-double six at least once in 24 throws with two
dice? (de Mere's Problem).
Solution. The probability of geuing a '6.' in a throw of die =1/6.
:. The probability of not geuing a '6 ' in a throw of die
=1 - 1/6 = 5/6 .
By compound probability theorem, the probability that in.4 throws of a die no
'6' is obtained =(5/6)4
Hence the probability of obtaining '6' at least once in 4 throws of a die
~ 1 - (5/6)4 =0·516
Now, if a trial consists of throwing two dice at a time, men the probability of
getting a 'double' of '6' in a trial =1/36 .
Thus the probability of not geuing a 'double of 6 ' in a trial =35/36.
The probability that in 24 throws, with two dice each, faO 'double of 6' is
obtained =(35/36)24
Hence the probability of geuing a 'double of·6' at least once in 24 throws
= 1 - (35/36)2A =0·491.
Since the probability in the fast case IS g~ter than the Pf9babili~y in the
second case, the result follows.
Exampie 4·18. A problem in Statistics is given to the three students A,B and
C whose chances of solving it are 1 / 2 ,3 /4 , and 1 /4 respectively.
What is ,the probability that the problem will be solved if all of them try
independentiy? [Madurai Kamraj Univ. B.Sc.,1986; Delhi Univ. B.A.,I991]
Solution. Let A, B, C denote the events that the problem is solved by the
students A, B, C respectively. Then
i 3 I
P(A)= 2' P(B) = 4 and P(C\= 4
The problem will be solved if at least one of them solves the problem. Thus
we have to calculate the probability of occ~ence of at least one of the three events
A , B , C , i.e., P (A u B u C).
P (A uB u C) =P (A)+P (B)+P (C)-P-(A ("\B)-P (A ("\ C)
- P (B ("\ C) + P (A n B nC)
=P (A) + P (B) + P (C) - P (A) P (B) - P (A) P (C)
- P (B) P (C) + P (A) P (B) P (C)
( ... A , B , C are independent events. )
1 3 1 1 3 3 1
= -+ -+ -
2 4 ~ 2 . 4 4·4
1 1 1 3 1
2 ·4 + 2 ·4 4
29
- 32
Visit : www.EasyEngineering.net
Visit : www.EasyEngineering.net
4·48 Fundamentals of Mathematical Statistics
= 1-(1-4)(1-~)(1-~)
29
- 32
Example 4·19. If A fiB =~, then show that ...(*)
P(A) ~ P (B)
[Delhi Univ. B.sc. (Maths Hons.) 1987]
Solution. We have
A = (A v (A fi B)
fi B)
=~ v(A fiB) [Using *]
=AfiB
A~B
::) P (A) ~ P(jj)
as desired..
Aliter. Since A fiB =~, we have A cB, which Uraplies thatP (A) ~P (Ii).
Example 4·20. Let A and B be two events such that
3 5
P (A) = - and P (B) = -
4 8
shOw that
(a) P(AuB~ ~ ~
(b) ~~P(A fiB) ~~
[Delhi Univ. B.sc. Stat (Hons.) 1986,1988]
Solution. (i) We have
A c (A vB)
::) P (A) ~ P(A v B)
::) ~~ P(AvB)
Visit : www.EasyEngineering.net
Visit : www.EasyEngineering.net
6+ ~ - 8 ~ P (A n B)
~ ~P (A nB)
...(ii)
( 1 -; ) x ( 1 -; ) = ( 1 ~; J; r =2, 3, 5, 7, '"
Hence the required probability that the two numbers chosen at random are
prime to each other is given by
P= ~ ( 1- ; J, where r is a prime number.
6
=2 (From trigonometry)
7t
Example 4·22. A bag contains 10 gold and 8 silver coins. Two successive
drawings of 4 coins ore made such that: 'm coins are repldced before thl! second
trial, (ii) thl! coins are not replaced before thl! second trial. Find thl! probability
that the first drawing will' give 4 gold and thl! second 4 silver ~oins.
[Allahabad Univ. B.Sc., 1987]
Solution. Let A denote the event of drawing 4 gold coins in the first draw and
Bdenote the event of drawing 4 silver coins in the second draw. Then we have to
fmd the probability of P ( A n B ). •
(i) Draws with replacement. If the coins drawn in the first draw are replaced
back in the bag before the second' draw then the events A and B are '~ndependent
and the required probability is given (using the multiplication rule of probability)
by the expression
P(AnB)=P(A).P(B) ...(*)
1st draw. Four coins can be drawn out of 10+8=18 coins in 18C4 ways, which
gives the exhaustive number of cases. In order that all these coins are of gold, they
Illust be drawn out of the 10 gold coins and this can be done in lOC4 ways. Hence
P (A) = lOC4 I 18C4
Visit : www.EasyEngineering.net
Visit : www.EasyEngineering.net
2nd draw. When the coins drawn in the first draw ¥e replaced before the 2nd
draw, the bag contains 18 coins. The probability Of drawing 4 silver coins in the
2nd draw is given by P (B) = 8C. / 18C••
Substituting in (*), we have
IOC4 8C 4
P(AnB)= - x-
18C. 18C4
(ii) Draws without replacement. If the coins drawn are not eplaced back before
(he second draw, then the events A and B are not independent and the required
probability is given by .
P(AnB)=P(A).P(B LA) ...(u)
As discussed in part-(i), P (A) = 10C. / 18C••
Now; if the 4 gold coins which were drawn in the fIrSt draw are not replaced
back, there are 18 - 4= 14 coins left in the bag and P (B I A) is the probability of
drawing 4 silver coins from the bag containing 14 coins out of which 6 are gold
coins and 8 are silver coinS.
Hence P (B I A) = 8C. / I·C.
Substituting in (**) we gl'!t
IOC4 8C 4
P(AnB)=. ~ x IIC
C4 •
Visit : www.EasyEngineering.net
Visit : www.EasyEngineering.net
_ (j) X .( ~1) ! _ ~
- Ci) X 7 - 195
Example 4·24. p is the probability that a man aged x years will die ina year.
Find the probability that out of n men AI. A2• ••• , A. each aged x, Al will die in a
year and will be thefirst to die. [Delhi Univ. B.sc., 1985)
Solution. LetEi, (i = 1,2, ... , n) denote the event that Ai dies in a year. Then
P (Ei) =p, (i =1,2•... , n) and P (Ed = 1 - p.
The probability that none of n men AI. A2• ... , A. dies in a year
=P (EI h E2 f"l ... f"l E.) =P (EI) P (E2) '" P (E,,)
(By compoilnd probability theorem)
= (l-p)"
. . The probability that at least one of AI. A2, ..., A", dies in a year
= 1 -: P (El f"l E" f"l ... f"l E,,) = I - (1 - p )"
The probability that among n men, AI is the flCSt to die is lin and since this
event is independent of the event that at le~t one man dies in a year, required
probability is
.; [ I - (I - p)" ]
Example 4·25.Thf! odds against ManagerX settling the wage dispute with the
workers are 8:6 and odds in favoUT of manager Y settling the same dispute are
14:16.
(i) What is the chance that neither settles the dispute, if they both try,
independently of each other?
(ii) What is the probability that the dispute will be settled?
Solution. Let A be the event lha.t the manager X will settle the dispute and B
be the event that the Manager Y will settle the dispute. Then clearly
P (A) = _8__= i => P (A) = 1 - P (A) = ~ = 1
8+6 7 14 7
~ 7 11 16 8
P (B) = 14 + 16 = IS => P ( ) = I - .P (B) = 14 + 16 = Is
The required probability that neither settles the dispute is given by :
P(Af"lIi)= P(A) x P(Ii'= i x ~= E-
. ' 7 IS lOS
[Since A and B are independent => A and Ii are also independent]
(ii) The dispute will be settled if at leaSt one of the managers X and Y settles
the dispute. Hence the required probability is given by:
=
P (A u B) Prob. [ At least one of X and Y settles the dispute]
=1 - Prob. [ None settles the dispute]
=1- P(Af"lB)= 1- E-=.!1..
lOS lOS
Visit : www.EasyEngineering.net
Visit : www.EasyEngineering.net
Example 4·26. The odds that person X speaks the truth are '3 :2 and the odds
that person Y speaks the truth are 5:3. In what percentage of cases are they likely
to contradict each other on an-identical poillt.
Solution. Let us define the events:
A : X speaks the truth, B : Y speaks.the truth
Then A and Ii represent the complementary events that X and Y tell a lie
respectively. We are given:
3 3 , 3 2
P(A)= - - = - ~ P(A)~ 1- - = -
3+ 2 5 :5 5
5
P(B)= - - = -
5 5 3
and ~ PUi)= 1- -= -
5+ 3 8 8 8
The event E thal' X and Y contradict ea~h other on an identical pomt can
happen in the following mutually exclusive ways:
(i) X speaks the truth and Y tells a lie, i.e., the event A n Ii happens,
(ii) X tells a lie and Y speaks the truth, i.e., the event An B happens.
Hen~e by addition .theorem of probability the required probability is given by:
Visit : www.EasyEngineering.net
Visit : www.EasyEngineering.net
Example 4:28. (a) A and B a/ternmeiy cut a pack of cards and·the pack is
shujJ1ed after each cut. If A starts and the game'is continued until one ctas a
diamond,' what are the respective chances of A and B first cutting a diamond?
(b) One shot isfiredfrom each oft/Ie three guns. E., E2, E3 .d~fjote-tlze events
that the target is hit by the firsJ. second and third gun respectively. If
P (E:) = 0·5, P (E'2) = 0·6 and P (E3) = 0·8 and. E\, £2, E3 are independent
events,find the probability that (a) exaGt/y one hit is registered,. (b) at/east two
hits are registered.
Solution. (a) Let 'E\ and E2, denote the events of A and £i cutting a diamond
respectively. Then
. 13 1 - - 3
P (E\) = P (E2) = - = - ~ P(E\) = P (E2) = -
52 4 . 4
If A starts the game, he can firs~ cl,lt the diamond in the following mutually
exclusive ways:
(i) E\ happens, (it) E\ n E2 n E; happens, (iii) E\ n E2 n £\ n E2 n E\
happens, and so on. "ence by addition theorem of probability-, the probabiltly 'p.
lhaL A first wins is given by
p = P (j) + P (ii)+ P (iii) + ..... .
= P (E\) + P (E\ n E2 n E\ ) + P (£~ n E2 n E\ n E2 n E\) + ...
= P (Ed + peEl) P (E2) P (E\) + l' (E\)'P (E2) P (E~) P (£2) P (E\) -I- •••
(By Compou/k.{ Probability Theorem}
1 331333 g 1
= 4" + 4"' 4"' 4" + 4"' 4"' 4"' 4"' 4" + ..... .
1
·4 4
=--9-=-=;
1- 16
The probapility thatB frrst cuts a diamond
4' 3
= 1- p= 1- - =-
7 7
(b) We are given
P (E\) = 0·5, P (E 2 ) = 0·4 and P (E3) = 0·2
Visit : www.EasyEngineering.net
Visit : www.EasyEngineering.net
Visit : www.EasyEngineering.net
Visit : www.EasyEngineering.net
EXERCISE 4 (b)
1. (a) Which function defines a probability space on S = (' elt e2. e3)
1 1 1
(i) P (el) = '4' P (e2) = '3' P (e3) = 2
{ii) P (ej),= ~. P (e2) = - ~ • P (e3) = ~
(,',',') P () 1 P ()
el = '4' 1 . P ()
e2 == '3 1 and
e3;:;: 2'
I 2
(iv) P (el) = 0, P (e2) = '3' P (e3) = '3
Ans. (i) No. (ii) No. (iii) No, and (iv) Yes
(b) Let S: (elo e2. e3, e,4) , and I~t P be a probability function on S ,
(ii) Find P(el) and P(e:z) if P(e3) =P(e4) :: ± and P(el) = 2P(e2). and
be events given,by
Visit : www.EasyEngineering.net
Visit : www.EasyEngineering.net
A man forgets the last digit of a telephone number, and dials the last digit at
~'~ndom, What is the probability of caHing no more than three wrong numbers?
(b) Define conditional probability and give its frequency interpretation. Show
that conditional probabilities satisfy the axioms of probability.
6. Prove the following laws, in each case asswning the conditional prob-
abilities being defined.
(a)p(£I£)=I, (b) P(eIlIF)=O
(c) IfEI r;;,£z, then E(EI I F)<P(Ezl F)
(d) p(EIF)= 1- p(EIF)
=
(e) P (£1 u £z IF) P (El IF) + P (Ez IF) - P (P (El n £z IF)
(f) If P (F) = I then P (E IF) = P (E)
(g) P (£ - F) = P (E) - P (E n F)
(h) If P (F) > 0, and E and F are mutully exclusive then P (E I F) 0 =
(i) If P (£ I F) == P (E) , then P (E I F) P (E) and P (£ I F) P (£)
= =
7. (a) UP<A)= a, P(8)= b,thenprovethatP(An8) ~ l-a,..b.
(b) If P (A) = a, P (8) =~, then prove that P (A I 8) ~ (a + ~ - l)/~.
Hint. In each case use P (A u 8) ~ 1
8. Prove or disprove:
(a) (i) If P (A 18) ~P (A), then P (8 IA) ~P (8)
(ii) if P (A) = P (8), then A = 8.
[Delhi Univ. B.Sc. (Maths Hons.), 1988]
(b) If P (A) =0, then A = ell
[QeJhi Univ. B.Sc. (Maths Hons.), 1990]
Ans. Wrong.
(c) ForpossibieeventsA,8, C,
(i) If P (A) > P (8), then P (A I C) > P (8 I C)
(if) IfP(AIC)~P(81c) and P(AI,C)~P(8IC)'
then P (A) ~ P (8). [Delhi Univ. B.Sc.(Maths Hons),1989]
(d)lfP(A)=O, then P(An8)=O.
[Delhi Univ. B.Sc. (Maths Hons.), 1986)
(e) (i) If P (A) = P (8) = p, then P (A n 8) ~pz
(iij If P (8 1:4) = P (8 I A), then A and 8 are independent.
[Delhi Univ. B.Sc. (Maths Hons.), 1990]
Visit : www.EasyEngineering.net
Visit : www.EasyEngineering.net
Visit : www.EasyEngineering.net
Visit : www.EasyEngineering.net
Visit : www.EasyEngineering.net
Visit : www.EasyEngineering.net
(ii) Y.ou will 'hit the target at least twice. Find also the probabilities of these
events. [Sardar Patel Univ. B.Sc., 1990)
15. (a) Suppose A and B are any two events and that P (A) = Pit P (B) = P2
and P (A II B) = P3' Show that the formula of each of the following
probabilities in tenns of Pit P2 and P3 can be expressed as follows:
(i) P (;;: u Ii) = I - P2 (it) P (~ II Ii) = I - PI - P2 + P3
(iii) P (A II B ) = PI - P3 (iv) P (A II B) = P2 - P3
---
(v) P (A II B) = I - P3 (vi) P (A u B) = I - PI + 1'3
(vii) P (A u I! ) = I - PI - P2 + P3 (viii) P [A II (A u B)] =P2 - P3
Visit : www.EasyEngineering.net
Visit : www.EasyEngineering.net
(b) Defects are classifcd ac; A, ,8 or C, and the following probabilities have been
detennined from av~il~ble production data :
P (A) =0·2{),·P (B) = 0·16, P (C) =0·14, P (A (") B) =0·08, P (A (") C) =0·05,
P (8 (") q = 0·04, and P (A (") 8 (") C) = 0·02.
What is the probability .that a randomly selected item of product will exhibit
at-least one type of defect? What is the probability that it exhibits both A and 8
defects but is free from type C defcct ? [Bombay Univ. B.Sc., 1991)
(c) A language class has only three students A, B, C and they independently
attend the class. The probat;>ilities of attendance of A, Band C on any given day are
1/2,213 and 3/4' respectively. Find the probability that the total number of
attendances in two consecutive days is exactly three.
(Lucknow Univ. B.Sc. 1990; Calcutta Univ. B.Sc.(Maths Hons.), 1986)
18. (a) Cards are drawn one by one from a full deck. What is the probability
that exactly 10 cards will precede the first ace. [Delhi Univ. B.sc.,1988]
48 47 46 39) 4 164
Ans. ( 52 x 51 x 50 x ... ~ 43 x 42 = 4165
(b) hac of (wo persoos tosses three frur coins. What is the probability that
they obtain the same number of heads.
P (Ai) =1 - ia1' ,=
,
1, 2, '" ,n .
Find the value of P (AI u A2 U A3 U ... u A.). (Nagpur Univ. B.Sc., 1987)
1
Ans. 1- ,. (>1+ lY:t
a
(d) Suppose the events A.. A2, ..., A" are independent and that
P (A;) =~1
I +
for 1 SiS n . Find the probability that none of the n events
occW's, justi~ng each step in your. calculations.
Ans. 1/( n + 1 )
20. (a) A denotes getting a heart card, B denotes getting a face card (King,
Queem or Jack), A and B denote the complementary events. A card is drawn at
Visit : www.EasyEngineering.net
Visit : www.EasyEngineering.net
Visit : www.EasyEngineering.net
Visit : www.EasyEngineering.net
(c) A husband and wife appear in an interview for two vacancies in the same
post The probability of husband's selection is In and- that of wife's selection is
1/5. What is the probability that only one of them will be selected·?
Ans. 2n [Delh,i tJniv. 8.Sc.,.1986]
24. (a) The chances of winning of. two race-horses are 1/3 and 1/6 respectiye-
ly. What is the probability that at least one will win when the horses are running
(a) in different races, and (b) in the same race?
Ans. (a) 8/18 (bJi(2
(b) A problem in statistics is given to three students whose chances of solving
itare 1(2,1/3 and 1/4. What is the probability that the problem will be solved?
Ans. 3/4 [Meerut Univ. B.Sc., .1990]
25. (a) Ten pairs .of shoes are in a closet. Four shoos are selected at random.
Find the probability that there will be at least one pair among the four shoes
selected?
IOC4 x' 24
Ans. 1 - - - -
:lDC4
(b) From 100 tickets numbered I, 2, '" , I 00 four are drawn at random. What
is the probability that 3 of them will bear number from 1 to 20 and the fourth will
bear any nwnber from 21 to l00?
:lDC) X 80CI
Ans,
IClOC4
26. A six faced die is so biased that it is twice as likely to show an even number
as an odd r.umber when thrown. It is thrown twice. What is the probability that the
sum of the two numbers thrown is odd?
Ans. 4/9
27. From a group of 8 children,S boys and 3 girls, three children are selected
al random. Calculate the probabilities tl)at selected group contains (i) no girl,
(ii) only one girl, (iii) one partjcular girl, (iv) at least one girl, and (v) more girls
than boys.
Ans. (i) 5(113, (ii) 15(113, (iii) 5(113, (iv) 23128, (v) 2f1..
28. If, three persons, selected at random, are stopped on a street, what. are the
probabilities that:
((I) all were born on a Friday;
(b) two were born on a Friday and the other on a Tuesday;
(c) none was born on a Monday.
Ans. (a) 1/343, (b) 3/343, (c) 216/343.
29. (a) A and B toss a coin alternately on the understandil)g -that the first who
obtains the head wins. If A starts, show that th~ir respective chances of winning are
213 and \/3.
(b) A, Band C, in order, toss a coil'. The first one who throws a head wins. If
A starts, find their respectivre chanc~s of winning. (Assume that the game may
Visit : www.EasyEngineering.net
Visit : www.EasyEngineering.net
continue indefinitely.)
ADs. 4/7. 2/7. 1/7.
(c) A man alternately tosses a coin and throws a die. beginning with coin. What
is the probability that he will get a head before he gets a '5 or 6'on die?
ADs. 3/4.
30. (a) Two ordinary six-sided dice are tossed.
(i) What is the probability that both the dice show the number 5.
(ii) What is the probability that 'both the dice show the same number.
-(iii) Given that the sum of two numbers shown is 8. find the conditional-
probability that the number noted on the first dice is larger than the number noted
on the second dice.
(b) Six dice.are thrown simultaneously. What is.the proliability tHat ai, will
show different faces?
3J. (a) A bag contains 10balls. two of which are red. three blue and five black.
Three balls are drawn at random from the bag. that is every l>aU has an equal chance
of being included in the three. What is the probability that
(i) the three balls are of different colours.
(ii) two balls are Qf the ~811le cQlour. and
(iii) the balls are all of the same colour?
Ans. (i) 30/120, (U) 79/120. (iii) 11/120.
(b) A is one of six hor~s entered for a race and is to be ridden by one of the
tWQ jockeys B and C. I.t is 2 to 1 that B rides A, in which case all the horses are
equally likely to win. with rider C, A's chance is trebled.
(i) Find the probability that A wins.
(ii) What are odds against A's winning?
[Shivaji Univ. B.Sc. (Stat. Hons.), 1992]
Hint. ProbabiJity of A's winning
=P ( B rides A and A' wins) + P (C rides A and A wins)
2 1 1 3 5
=-x-+-x-=-
3 6 3 6 18
.. Probability of A's losing-= 1- 5/18 = 13/18.
HenceoddsagainstA'swinningare: 13/18:'5/18, i.e., 13:5.
32. (a) Two-third of the students in a class are boys and the rest girls. it is
known that the probability of a girl getting a first class is 0·25 and that of boy getting
a first class is 0·28. Find the probability that a student chosen at random will get
farst class marks in the subject.
ADS. 0·27
(b) You need four eggs to make omeJettes for breakfast You find a dozen eggs
in the refrigerator- but do not realise that two of these are rotten. What is the
Probability that of the four eggs you choose at random
(i) norie is rotten,
Visit : www.EasyEngineering.net
Visit : www.EasyEngineering.net
Visit : www.EasyEngineering.net
Visit : www.EasyEngineering.net
P3 = Prob. of drawiJ;lg one red and one blue ball =.[ (r + b)2:;'b _I)].
Now PI =5 P1 and P3 =6 P1
.. r(r-I).=5b(b- H and 2br=6b(b-l)
Hence b =3 and r =6.
39. Three newspapers A, 8 and C are published in a certain city. It is estimated
from a survey that 20% read Ai 16% read 8, 14% read C, 8% readl A and 8, 5% read
A and C, 4% read Band C and 2% read all the three newspapers. What is the
probability that a nonnally cho.sen person
(i) does not read any paper, (iO does not read C
(iii) reads A but not 8, (iv) reads only one of these papers, and
(v) reads only two of these papers.
Ans. (i) 0·65, (ii) 0·86, (iii) 0·12, (iv) 0·22. (v)'O·I1.
40. (0) A die is thrown twice. the event space S consisting of the 36 possible
pairs of outcomes (a,b) each assigned probability 1/36. Let A, 8 and C denote the
following events :
A={ (a,b)l a is odd), 8 = (a,b) I b is odd}, C = {(a,b) I a + b is odd.}
Check whether' A, B and C are independent or independent in pairs only.
[Calcutta Univ. B.Sc. Hons., 1985]
Visit : www.EasyEngineering.net
Visit : www.EasyEngineering.net
(b) Eight tickets numbered 111. 12.1. \22. 122.211.212.212.221 are plaCed
in a hat and stirrt'ti. One of them is then drawn at random. Show that the-event A :
"t~e first digit on the ticket drawn wilrbe 1". B : "the second digit on the ticket
drawn will be L" and C : lithe third digit on the ticket drawn will be 1". are not
pairwise independent although
P (A n Bn C) = P (A) P (B) P (C)
41. (a) Four identical marbles marked 1.2.3 and 123 respectively are put in
an urn and one is drawn at random. letA. (i =1.2.3). denote the event that the
number i appears OJ) the drawn marble. Prove that the events AI.A 2 andA l are
pairwise independent.but not mutually independent.
[Gauhati Univ. B:Sc. (Hons.), 1988]
1
Hint. P'(AI) =2" =P (A2) =P (Al )'; P(Af A2) =P (AI Al ) =P (A2Al) =41
'I
P (A I A2 A3) = 4'
(b) Two fair dice are thrown independently. Define the following events :
A : Even number on the first dice
B : Even number on the second dice.
C : Same number on both dice.
Discuss the independence of the events A. B and C.
( c) A die is of the shape of a regular tetrahedron whose faces bear the numbers
111. 112. 121. 122. AI.A2.Al are respectively the events that the first two. the
last t~o and the extreme two digits are the same. when the die is tossed at random.
Find w~ether or not the events AI, A2• Al are (i) pairwise independent. (ii) m~tually
(i.e. completely) independent. Detennine P (AI I A2 Al ») and explain its value by
by argument. [CiviJ Services (Main), 19831
42. (a) For two events A and B we have the following probabilities:
P (A)=P(A I B)=± and P (B I A)=4.
Check whether the following statements are tr,ue or false :
(i) A and B are mutually exclusive. (ii) A and B are independent. (iii) A is a
subevent of B. and (iv) P (X I B) -= ~
Ans. (i) False, (ii) True., (iii) False, and (iv) True.
(b) Consider two events A and B such that P (A)·= 114. P (B I A) 112. =
=
P (A I B) 114. For each of the following statements. ascertain whether it is true
or false:
(i) A is a sub-event of B. (ii) P (A IIi) =3/4.
(iii) P (A I B) + P (A IIi) =1
43. (a) Let A and B be two events such that P (A) = 4'3 and P (B)
5
= "8'
Visit : www.EasyEngineering.net
Visit : www.EasyEngineering.net
Show that
(i) P (A v 8) ~ ~. (ii) ~:5; P (A.n 8,) :5; %. and (iii) i:5; P (A n B) :5; ~ •
[Coimbatore Univ. B:E., Nov. 1990; Delhi Univ. n.sc.(Stat. Hons.),1986)
(b) Given two events A and B. If the odds against A are 2 to 1 and those in
favour of A v B are 3 to I, shOw that
152:5; P (B) 5 ~
Give an example in which P (B) = 3/4 and one in which P (B) = 5/12.
44. Let A and B be events, neither of which has probability zero. Prove or
disprove the following events :
(i) If A and B are disjoint, A and'B are independent.
(ii) If A and B are independent, A and 'B are disjoint.
(b) There areM urns numl5cred 1 toM andM balls numbered 110M. ThebalJs
are inserted randomly in the urns with one ball in e~ch urn. If a ball is put into the
urn bearing the same number as the ball, a match is said to have occurred. Find the
probability thllt t:lo match has occurred. . [Civil Services (Main), 1984J
Hint. See Example 4· 54 page 4·97.
47. If n letters are placed at random in n correctly addressed envelopes, find'
the probability that
(i) none of the letters is placed in the 'correct envelope,
Visit : www.EasyEngineering.net
Visit : www.EasyEngineering.net
Visit : www.EasyEngineering.net
Visit : www.EasyEngineering.net
I 2
L ••_ _---1[1-1---.---11 ~f---:-_. R
3
1 I'r--J
4
Ans. p2 (2 - p\
4·9. Bayes Theorem. If EI. E2• ... , E. are mutually disjoint events with
p (E,) 1:: O. (i = I. 2•... , n) then for any orbitrary event A which is a subset of
fa
Visit : www.EasyEngineering.net
Visit : www.EasyEngineering.net
11 11
= r. P (Ei I A) P [C I (EinA)]
i= I
Substituting the value of P (Ei I A) from (*). we get
11
11
r. P (Ei) P (A lEi)
i= 1
Rtmark. It may happen that die materialisation of the event Ei makes C
i.ridcQendenl of A. then we have .
. P(C I Ei nA)= P(C lEi).
and the abo~e.Jormula reduces to
·11
l: P(Ei) P (A i E i) P (C I Ei)~
P (C I A) = ..:...i=.....:l~_____________
•..(4·12 c)
11.
l: P (Ei) P (A lEi)
i= I
l11e event C can be considered in r~gard to A. as Future Event.
Visit : www.EasyEngineering.net
Visit : www.EasyEngineering.net
4·71
Example 4·30. In 1?89 there were three candidates for the position 0/
principal- Mr. Challerji, Mr. Ayangar and Dr. Sing/!., whose chances 0/ gelling
the appointment are in the proportion 4:2:3 respectively. The prqbability that Mr.
Challerji if selected would introduce co-education in the college is 0·3. The
probabilities 0/ Mr. Ayangar and Dr. Singh doing the same are respectively 0-5
and 0·8. What is the probability that there was co-education in the college in 199O?
(Delhi Univ. B.Sc.(Stat. Hons.), 1992; Gorakhpur Univ. B.Sc., 1992)
Solution. Let the events and probabilities be defined as follows:
A : Introduction or co-education
EI : Mr. Chauerji is selected as principal
E2 : Mr. Ayangar is selected as principal
E3 : Dr. Singh is selccted as principal.
Then
4 2 3
P (E1) = "9' P (E2) = "9 a~d P (E3) ±; "9
P (A'I E1) =,l
10'
P (A I E~) = 2-
10
and P (A 'I E3)·=!-
10
.. P (A) =i' [(A n E 1) u (A n
E 2), u (A n £3)]
= P(AnEd + P(AnE,.) + P(l\nE3)
= P (E 1) P (A. 1 E 1) + P (E2) P (A 1 E 2) :+- P (E3) P (A I ~3)
4 3 2 5 3 8 23
= "9 . 10 + "9 . 10 + "9 . 10 = 45
..:xample. 4·31. The contents·o/urns I, II and 11/ are as/ollows:
I white, 2 black and 3 red balls,
e white, 1 black and 1 red balls, and
<I white, 5 black and 3 r(;~.balls.
One urn is chosen at random and two balls drawn. They happen Jo be white
and red. What is the probability .thatthey come /rpl!l urns I, II or 11/ ?
.[Delhi ·Univ. B.Sc. (Stat. Hons.), 1988)
Solution. LetE\, E l • and E3 denote the events that the urn I, II and III is chosen,
respectively, and let A be the event that the two balls taken from the selected urn
are white and red. Then
-
P (E 1) = P (E = P (E3) = '31
2)
P (A 1 E 1) = 1 x 3 =.! P (A 1 £2-) = 2 x 1 = .!
6C2 5• 'C2 3'
4x 3 2
and P (A 1
£3) = 12C2 = IT
Visit : www.EasyEngineering.net
Visit : www.EasyEngineering.net
Hence'
'p (El I A) = ;. ~E2) P (A I E 2)
L P (E i) P (A lEi)
j", 1
Similarly
30
118
Visit : www.EasyEngineering.net
Visit : www.EasyEngineering.net
Solution. Let EJ. f-z and E, denote the events that a bolt-selected at random
is manufactured by the machinesA, B aM C respectively l,Uld let E denote the event
of its being defective, Then we have
P (E 1) = 0'25, P (£z) = 0·35, P (E3) = 0·40
The probability of drawing a defective .bolt manufactured by machine A is
f(E I E1)=0·05. ~
Similarly, we have
P (E I El ) =0·04, and f (E I E3) =0·02
Hence the probability t~at a defective bolt selected-at random is manufactured
by machine A is given by
P (El I E) = : (E 1) f (E lEI)
~ P (Ej ) P (£ I E j )
i= I .
0·25 x 0-05 125 25
= =-=-
0-25 x 0-05 + 0·35 x 0·04 + 0·40 x 0·02 345 69
Similarly
p(£zl'E)= .. 0·35 x 0·04. 140 28
0·25 x 0·05 + 0·3? x 0·04 + 0·40 x 0·02 =345 =69
and
P (E3 , E) = 1 - [P (El , E) + P (£1 , E)] = 1 _ 25 _ :28 :: ~
69 69 69
This example illustrates one of the chief applications of Bayes Theorem.
EXERCISE 4 (d)
1. (a) State and prove Baye's Theorem.
(b) The set of even~ Ai , (k = 1,2, ... , n) are (i) exhausti~e and (ii) pairwise
mutually exclusive. .If for all k the probabilities P (Ai) and f (E I Ai) are known,
calculate f (Ail E), where E is an arbitrary evenL Indicate where conditions OJ and
(ii) are used.
(c) 'f.he events E10 El , ... , E.. are mutually exclusive and, E =El u el U .. ,
u E... Show. that if f (A , E j ) = P (B , Ej) ; i = 1, 2, ..., n, then P(A' E) =
P(B', E). Is this conclusion true if the events,Ei are not mutually exclusive?
[Calcutta Univ.• B.Sc. (Maths Hons.), 1990)
(d) What are the criticisms ~gai~t the use of Bayes theorem in probability
theory. [Sr:i. Venketeswara Univ. B.Sc., 1991)
(e) Usillg the fundamental addition and multiplication rules Q( prob<lbility,
show that
__ P(B)P(AIB)
P (B I A) - P (8) P (A I B) + P (B) P (.4 I B)
Where B is the event complementary to the eventB.
[Delhi Univ. M.A. (.:con.), 1981)
Visit : www.EasyEngineering.net
Visit : www.EasyEngineering.net
2. (a) Two groups are competing for the positions on the Board of Directors
of a corporation. The-probabilities that the first and second groups will wiii are 0·6
and 004 respectively. Furthermore, if the first group wins the probability of
introducing a new product is 0·8 and the corresponding probability if the second
group wins is O· 3. Whl\t is the probability that the new product will be introduced?
Ans. 0·6 x 0·8 + 04 x 0·3 =0·6
(b) The chances of X, Y, Zbecoming'managersol a certain company are 4:2:3.
The plOoabilities that bonus scheme will be introduced if X, Y, Z become managers.
are O· 3. (f·5 and 0·8 respectively. If the bonus scheme has been introduced. what is
the probability that X is appointed as the manager.
Ans. 0·51
(c) A restaurant serves two special dishes. A and B to its customers consisting
of 60% men and 40% women. 80% of men order dish A and the rest B. 70% of
women order dish B and the rest A. In what ratio of A to B should the restaurant
prepare the two dishes? (Bangalore Univ. B.Sc., 1991)
Ans. P (A) =P [(A n M) u (,4. n W)] = 0·6 x 0·8 + 004 x 0·3 = ()'6
Similarly -P (B) =0·4. Required ratio =0·6 : 004 =3 : 2.
3. (a) There are three urns having the following compositions of black and
white balls.
Urn 1 : 7 white. 3 black balls
Urn 2 : 4 white. 6 black balls
Urn 3-: 2 white. 8 black balls.
One of these urns is chosen at random with probabilities 0·20. ()'60 and 0·20
respectively. From the chosen urn two balls are drawn at random without replace-
ment Calculate the probability that both these balls are white.
Ans. 8145. (Madurai Univ. B.Sc., 1991)
(b) Bowl I contain 3 red chips and 7 blue chips. bowl II contain 6 roo chips
and 41blue chips. A bow I is ·selected at random and then 1 chip 'is drawn from this
bowl. (i) Compute the probability that this chip is red. (ii) Relative to the hypothesis
that the chip is red. find the conditiona! probability that it is drawn from boWl II.
[Delhi Univ. B.Sc. (Maths 80ns.)1987]
(c) In a (actory machines A and B are producing springs of the same type. Of
this production. machines A and iJ produCe 5% and 10% defective springs.
respectively. Machines A and B produce 40% and 60% of the total output of the
factOry. One spri~g is selected at random and it is found to be defective. What is
the possibility that this defective spring was pfuduced by machine A ?
. [Delhi Univ. M.A. (Econ.),1986]
(d) Urn A con'tains 2 white. 1 blac~ and 3 red balls. urn B contains 3 white. 2
black and 4 red balls and urn C con~ns 4 white. 3 black and 2 red balls. One urn
is chosen at random and 2 ,balls are drawn. They happen to be'red and black. What
Visit : www.EasyEngineering.net
Visit : www.EasyEngineering.net
P (A lEI) = l~ ~d P (El I A) = ;0
(c) It is known that an urn containing ·a1together 10 balls was' filled in, the
following manner: A coin was tossed. 10 times, and accordiJ]g a~ it showed heads
Or tails, one white or one black ball was put into· the ul1.l.·Balls are dfawnJrom lhis
Visit : www.EasyEngineering.net
Visit : www.EasyEngineering.net
urn one at a time, to times in succession (with replacement) and everyone turns
out to he white. Find the chance that the urn contains nothing but white balls.
Ans. 0·0702. .
5. (a) Frqm a vessel containing 3 white and 5 black balls, 4 balls are
transferred into an empty vessel. From this vessel a ball is drawn and is found to
be white. What is the probability that out of four balls transferred. 3 are white
and I black. [Delhi U~i. B.Sc. (Stat. 1ions.), 1985]
Hint. Let the five mutually exclusive events for the four balls transferred be
Eo. E It E 2 , E 3 , and E 4• where E; denotes the event that i white balls are
transferred and let A be the event of drawing a white ball from the new vessel.
sC4 3C I x 5C 3 3C 2 X 5C 2
Then P (~o) =8C4 ' P(E I') 8C4 ' P (E 2) = 8C4
3C 3 X SCI
P (E) = 8C4 and P (E4 ) = 0
I 2 3
Also P(A I'Eo) =0, P (A lEI) =4' P (A I E 2) =4' (A I E3) =4'
I
and P (A l'E4 ) = I·. Hence P(E31 A) = =;.
(b) The contents of the urns I and 2 are as follows:
Urn 1 : 4 white and 5 black balls.
Urn 2 : 3 white and 6 black balls.
One urn is chosen at random and a ball is drawn and its colour noted and
replaced back to the um. Again a ball is drawn from the same urn, colour noted
and replaced. The process is repeated 4 times and as a result one ball of white
colour and three balls of black colour are obtain, 1. What is the probability that
the urn chosen was the urn 1 ? (Poona Univ. B.E., 1989)
Hint. P(E I ) = P (E2) 1F 1/2,
P (~ lEI) 4/9, = I - P (A lEI) 5/9=
P (A I E 2 ) = 1/3, 1 - P (A I E 2 ) = 2/3
The probability that the urn chosen was the urn I
! 1. (~)3
2· 9 9
=------------------------
! 4 (~)3 L 1, .(~)3
2· 9· 9 + 2· 3· 3
(c) There are five urns numbered I to 5. Each urn contains to balls. The ith
urn has i defective balls and 10 - i non-defective balls; i = 1,2, ... 5. An urn is
chosen at random and then a ball is selected at random from that urn. (i) What is
thc probability that a defective ball is selected?
(ii) If the selected ball is defective, find the probability that it came from
urn i. (i = 1,2, ... , 5). [Delhi Univ. B.Sc. (Maths Hons.), 1987]
Hint.: Define the following events:
E;: ith urn :is selected at random.
Visit : www.EasyEngineering.net
Visit : www.EasyEngineering.net
For example, the probability that the defective ball came from 5th urn
= (5/15) = 1/3.
6. (a) A bag contains six balls of different colours and a ball is drawn from it
at random. A speaks truth thrice out of 4 times and B speaks truth 7 times out of 10
times. If both A and B say that a ,ed ball was drawn, find the probability of their
joint statement being true.
[Delhi Univ. B.Sc. (Stat. Hons.),1987; Kerala Univ. B.Sc.I988]
(b) A and B are two very weak students of Statistics and their chances of
solving a problem correctly are 1/8 and 1/12 res~tively. If the probability of their
making a common mistake is 1/1001 and they obtain the same answer, find the
chance that their answ~r is correct. [Poona Univ. B.sc., 1989]
A R d Pr bab 'I' I;S x 1112 13
ns. eq. 0 I Ity = 1111 x ilI2 + (l - I;S) • (l ~ ilI2) . ilIool 14
7. (a) Three bOxes, practicaHy indistinguishable in appearance, have two
drawers·each. Box l.contaiils a gold 'coin in one and a sit ver coin in the other drawer,
boX'II contains a gold coin in each drawer and box III contains a silver coin in each
drawer. One box is chosen at random and one of its drawers is opened at random
and a gold coin found. What is the:probability that the other drawer contains a coin
of silver? (Gujarat Univ. B.Se., 1992)
Ans. 113, 113.
(b) Two cannons No. I and 2 fire at the same target Carinon No. I gives.on
an average 9 shots in the time in which Cannon No.2 fires 10 projectiles. But on
an average 8 out of 10 projectiles from Cannon No. 1 and 7outofIO from Cannon
No. 2 ~trike the target. [n the course of shooting. the target is struck by one
projectile. What is the probability of a projectile which has struck the target
belonging to Cannon No.2? (Lueknow Univ. B.Sc., ,1991)
Ans. 0·493
(c) Suppose 5 mep out of 100 IlDd 75 women Ol,lt of 10.006 are colour.blind.
A colour blind person is chosen at random. What is the probability of I)is being
male? (Assume.males and females to be in equal number.)
Hint. £1 = Person is a male, E2 = PersOn is a female.
Visit : www.EasyEngineering.net
Visit : www.EasyEngineering.net
Visit : www.EasyEngineering.net
Visit : www.EasyEngineering.net
T.B. on the basis of the X-ray test, what is the probability of his.actually having a
T.B.? (Nagpur Univ. B.E., 1991)
Ans. 0·97
11. A certain transistor is manufactured at three factories at Bamsley, Bradford
and BrisLOl.lt is known that the Bamsley facLOry produces twice-as many transisLOrs
as the Bradford one, which produces the same number as the BrisLOI one (during
the same period). Experience also shows that 0·2% of the transistors produce4 at
Bamsley and Bradford are faulty and so are 0·4% of those produced at BrisLOI.
A service engineer, while maintaining an electronic equipment, finds a defec-
tive transistor. What is the probability that the Bradford facLOry is to blame?
(Bangalore Univ. B.E., Oct. 1992)
12. The sample space CO!1Sists of integers from -I to 2n which are assigned
probabilities proportionalLO their logarithms. Find the probabilities and show tbat
the conditional probability of the integer 2, given that an even integer occurs, is
log 2
[n log 2 + log (n ! ) ] (t.ucknow Univ. M.A., 1992)
(Hint. Let Ej : the event that the integer 2i is drawn, (i = 1,2, 3, ... , n ).
A : the event of drawing' an even integer.
11
p(E.IA)= I+(:-I)P
14. DieA has four red and two, white faces whereas dieB has two red and four
White faces. A biased coin is flipped once. If it falls heads, the game d,'ntinues by
Visit : www.EasyEngineering.net
Visit : www.EasyEngineering.net
Visit : www.EasyEngineering.net
Visit : www.EasyEngineering.net
probability that a point selected'at random in a given region will lie in a,specified
part of it, the classical definition of probability is modific<l and extended to what
is called geometrical probability or probability in continuum, In this case, the
general 'expression for probability 'p' is given by
_ Measure of specified part of the region
p- Measure of the whole region
where 'measure' refers to the length, area or volume of the region if we are dealing
with one, two or three dimensional space respectively.
Example 4·34. Two points are taken at random on the given straight line of
length a. Prove that the probability of their distance exceeding a given length
P(lx-yl>e)= :1
(a 'e)2 (
I-~)
ef
Visit : www.EasyEngineering.net
Visit : www.EasyEngineering.net
Visit : www.EasyEngineering.net
Visit : www.EasyEngineering.net
.::.. .[_<,;:;..,al2:;<.,.L....::.x_d---,Ydx_"
a a-x
= l [~- (~-
a
x )] dx
f f dydx f (a -x) dx
o 0 o
rf I~/~
-1-(a2-x)2'1~ = a2/2 = '4
a2/8 1
where k is a constant.
Os; Y s; a12, ...(*)
a
1
The needle will jnt~lsect one of the
lines if the distance of its cenlre from me
line is less than ~ I sin~, i.e., the required
event can be represented by the inequality
I
t
0< y < I sin ~ . Hence.the required probability p is given by
It ('~.)12
J J
-0 0
f(Y, ~) dy df!>
1= . It -alZ
J J f(y,~)djiJ~
o 0
It
~J0 sin~d~
= (a/2) .1t .
- a1t l-cos~I~= an
21
Visit : www.EasyEngineering.net
Visit : www.EasyEngineering.net
EXERCISE 4 (e)
1. Two points are selected at random in a line AC of length 'a' so as to lie on
the opposite sides of its mid-point O. Find the probability that the distance between
them is less than a/3 .
2. (a) Two points are selected at random on a line of length a. What is the
probability that .10ne of three sections in which the line is thus divided is less than
al41
Ans. 1116.
(b) A rectilinear segment AB is divided by a point C into two parts AC=a,
CB=b.PointsXand Y are taken at random onAC and CB respectively. What is the
probability thatAX,XYand BY can form a triangle?
(c)ABG is a straight line such thatAB is 6 inches and BG is S' inches. A pOint
Y is chosen at random on the BG part of the line. If'C lies between Band G in such
a way that AC=t inches, find
(i) the probability that Y will lie' in BC.
(ii) the probability that'Y will lie in CG.
What can you say about the sum of these probabilities?
(d) The sides of a rectangle are taken at random each less than a and all lengths
are equally likely. Find the chance that the diagonal is less than a.
3. (a) Three points are taken at random on the circumference of a circle. Find
the chan~ that they lie on the same semi- circle.
(b) A chord is drawn at random in a given circle. Wliat is the probability that
iris greater than the side of an equilateral triangle inscribed in that circle?
(c) Show that the probability of choosing two points randomly from a line
segment of length 2 inches and their being at a distance of at least I inch from each
other is 1/4. [Delhi Univ. M.A. (Econ.), 1985]
4. A point is selected at random inside a circle. Find the probability that the
point is closer to the centre of the circle than to its circumference.
S. One takes at random two points P and Q on a segment AB of length a
(i) What is the prooabiJity for the distancePQ being less than b ( <a )1
(ii) Find the chance that the distance between them is greater t~an a given
length b.
6. Two persons A and B, make an appointment to meet on a certain day at a
certain place, but without fixing the time further than that it is to be between 2 p.m.
and 3 p.m and that each is to wait not longer than ten minutes for the other.
Assuming that each is independently equally likely to arrive at any time during the
hour, find the probability that they meet.
Third person C, is to be at the same place from 2·10 p.m. until 2·40 p.m. on
the same day. Find the probabilities of C being present when A and B are there
together (i) When A and B remain after they meet, Oi) When A and B leave as soon
as they meet.
Visit : www.EasyEngineering.net
Visit : www.EasyEngineering.net
Hint. Denote the times of arrival of A by x and of B by. y. For the meeting to
take place it is necessary and sufficient that
Ix-yl<lO
We depict x and y as Cartesian coordinates in the plane; for the scale unit we
take one minute. All possible outcomes can be described as points of a ~qujife with
side 60. We shall finally get [cf. Example 4·34, with a == 60, c =lO]
P [I x - y 1< lO] = 1 - (5/6)~ = 11136
7. Tile outcome of an experiment are represented by points 49 the square
bOunded by x = 0, x = ~ and y = 2 in the ~-plane., If the probability is distributed
~niformly, determine the probability that Xl + l > 1
Hint.
Required probability P (£) = J ~ dx dy = 1 - J
~ dx dy
E E'
where E is the region for whiCh Xl + 'l:> 1 and' E' is the region for which
Xl+ l~ 1.
1 1
4P (£) =4 - J Jdx dy =3 P(£)=-
3
4
o 0
8. A floor is paved with tiles, each tile, being a parallelogram such that the
distance between pairs of opposite sides are a and b respectively, the length of the
diagonal being I. A stick of length c falls on the floor parallel to the diagonal. Show
that the probability that il will lie entirely;on one tile is
(1-7 r
If a circle of diameter d is thrown on the floor, show that the probability that
it will lie on one tile is
(I-~J (I-~)
9. Circular discs of radius r are thrown at random on to a plane circular table
of radius R which is surrounded by: a border of uniform width r lying in'the same
plane as the table. {f the discs are thrown independently and at random, end N stay
on the table, show that the probability that a fixed point on the table but not ~ the
border, will be covered is
1- [1- (R:d J
SOME MISCELLANEOUS EXAMPLES
Example 4·38. A die is loaded in' such a manner thatfor n=l. 2, 3. 4. 5./6.the
probability of the face marked n. landing on top when the die is rolled is propor-
tional to n. Find the probability that an odd number will appear on tossing the d,e.
[Madras Univ. D.SIe. (Stat. Mafn),1987]
Visit : www.EasyEngineering.net
Visit : www.EasyEngineering.net
Visit : www.EasyEngineering.net
Visit : www.EasyEngineering.net
Example 4·42. A car is parked among N cars in a row, not,ilt either end. On
his return the owner finds that exactly r of the N pJaces are still occupied.Whaiis.
lhe probability that both neighbouring places are empty?
Solution. Since the owner finds on return lhat exactly r of the /Ii places
(including.Qwner's car) are occupied, the exhaustlve number of cases for such an·
arrangement is N-1C,_1 [since the remaining r!... 1 cars are to be parked in 'the
remainingN - I places and thiscan·bedone in N-1C,_1 ways].
Let A denote the event that both the neighbouril)g places to owner's car are
empty. This requires the remaining (r - 1) cars to be parked in 'the remaining
N- 3 places and hence the num6er of cases favourable to A is N- 3C;_I. Hence
N-'3' , '
P(A) = C,_~ = (N-r)(N-r-l)
N-1C,_1 (N' - I)(N - 2)
Exam pIe 4·43. What is the probability thai at least two out of n people have
lhe same birthday? Assume 365 days in a year and that all days are equally likely. ,
Visit : www.EasyEngineering.net
Visit : www.EasyEngineering.net
Visit : www.EasyEngineering.net
Visit : www.EasyEngineering.net
Theory of Probability
Simil¥ly the number ,of 5.digited numbers ending with 24 and 32 each is 4.
Hence the total number of favourable cases is
3 x 3 ! + 3 x 4 = 18 + 12 ~ 30
. ed probabT
Hencerequlf 30 J6
llty= 96= 5
Visit : www.EasyEngineering.net
Visit : www.EasyEngineering.net
Solution, The score n can be reached in the following two Jrlutually exclusive
ways:
(i)By throwing a tail when score is (n - 2), and"
(ii)By throwing a head when score is (n - I),
•HenCe by addition the<,>~m of probability, we get
=
P~ P.(i) + P (ii) =t ,P~-2 +! 'P~-I =t (P~-I + P~-2 ~
To find P~ explicitly, (*) may. be re-written as
P~ + 2.1 P~-I =.P,;-I + 2.I P~-2
I
= Pl'+'2 P1
S~nce the score 2 can be obtained as
(i)Head in fU'St throw and head in 2nd throw,
"(ii)TaiI in tbe first throw, we have
111113 . 1
P2=- -+-=-+-=- and obvIOusly PI =-
2'2 2 4 2 4 2
Henc~, from (u), we get
1 3 1 1- 2 1 2 1 2
P~ + 2. P~ -I = 4 + 2.' 2. = 1 = 3" + 3" = '3 + 2. ' 3"
P~ - i =(- ~) (p. - 1- ~)l
PN - I - i =~ - t) (p. - 2 -:~)
P2 - t = ( -~) (PI - i)
Multiplying all the above equations. we get
P~ -i=(-~)1I-1 (PI-i>
= (_.!.)N-I (!_~)= (-I)" .1 !
:: :: 3 • 2" ' '3
2 (- I)" I
p.= 3'+ - 2" '3'I
= .!3 [2 + ( - 1)" 1.]
2"
Example 4·47. A coin is tossed (m+n) times, (mn). Show that the probability
. L __ J _ ' n + 2
of at Ieast m consecutive m:uu3 IS "Z" + 1 '
Visit : www.EasyEngineering.net
Visit : www.EasyEngineering.net
= (~J
P (E2) =P [Tail in the first toss, followed by m cOJ)~utive'heads and
head or tail in the nexU
= ~ (~J = 2}+ I
In general,
P (E,) =P [tail in the (r - I)th trial followed by m consecutive heads
J-"
and head or tail in the next]
= '21 ('I'2 = 2""1-'
1" .
-V r = 2, 3, ... , n + 1.
Substituting in (*),
R 'ed probabT
eqUlC
1 n 2+n
llty:: 2'" + 2"'+ 1= 2"'+ I
Examp~e 4·48. Cards are dealt one by one from a well-shuffled pgck until an
ace appears. Show that the probability that exactly n cards are dealt before the
first ace appears is
4(51 - n) (50 - n) (49 - n)
52.51.50.49 [Delhi Univ. B.Sc. 1992]
Solution. Let Ei denote the event diat an ace appears when the ith card is
dealt. Then the required probability 'p' is given by
p:;= P '[Exactly n cards are dealt before the first ace appears]
= P [The first ace appears at the (n + l)th dealing]
= P (E I fi E2 fi E3 fi .,. fi EM-I fi EM fi Eu l ) ,
= P (EI ) P (E2 1E I ) P (E3 lEI fi EJ ...
x P (EM I EI
, fi E2 fi . '.' fi EM-I) X P (E. + I I EI fi E2 fi ... fi E.)
I ••• (*)
Now
P (EI ) = ~~
- - 47
P (E2 I EI ) = 51
Visit : www.EasyEngineering.net
Visit : www.EasyEngineering.net
- - - 4
P(E~_IIElnE2n ... nE~_J= 52-(n-2)
- - - - 5O-n
P(E~_IIEI nE2n ... nE._J'~ 52-(n-2)
- - - 4
p ( E~ I EI n E2 n ... n E~ -I) = 52 ~ (n - l)
- - - - 49- n
P (E~ I EI n E2 n ... n E._I) = 52 _ (n _ 1)
- - - 4
P ( Eu d EI n E2 n ... n E~) = 52 _ n
, :- Hence. from (*) we get
~ -[48 47 46 45 44 43 _ 52 - n
p= 52x5tx50x49x48x47x ... x 52 - (n - 4)
.x 51 "
- n x 50 - n x 49 - n x - 4-]
52- (n- 3) 52- (n- 2) 52- (n- I) 5+- n
_ (51 - n)(50 - n)(49 - n) 4
- 52x51 x50x49
Example 4-49. If/our squares are cliosen at random on d chess-board,find
the chance 'that they slwuJd be in a diagonal line.
[Delhi Univ. B.Sc. (Stat. Hons.), 1988]
Solution. In a chess-board there are 8 x 8 = 64 squares as shown in the
followim~ diagram.
Let us consider the number of ways in
A which the 4 squares selected at random are
Al ~~-+--~~~-+-+~ in a diagonailine parallel to AB. Consider
AI I~~~__~~~-+-+~ the II ABC. Number of ways in which 4
A3 selected squares are along the lines ~ B••
~ ~~~~~--~~~
.A,B,. A2B2. AIBI and AB are ·C4• 5C••
"C•• '7 C4 and sC. respectively.
Similarly. in llABD there are an
equal number of ways of selecting 4
squares in a diagonal line parallel to AB.
Hence, total number of ways in which
the 4 selected squares are in a diagonal
line parallel to AB an
2 (·C. + 'C4 + 6C. + 7C.) + ·C••
Visit : www.EasyEngineering.net
Visit : www.EasyEngineering.net
Visit : www.EasyEngineering.net
Visit : www.EasyEngineering.net
Solution. Since each of the two 4if:e can show anyone of the six faces 1,2,3,
4,5, 6, we get:
P(A) = 3 x 6 = .! [.: A= (1,3,5) x (1,2,3,4,5,6)]
36
P(B) = 3 x 6 =
2
.! [ .: B
-
= (1,2,3,4,5,6) x
,.
(1,3,5) ]
36 2
The sum Qf points on two dice will be 0<14 if one shows odd number and the
other shows even number. Hence favourable cases for C are :
(1,2), (1,4), (1,6); (4, I), (4,3), (4,5)
(2, I), (2,3), (2,5); (5,2), (5,4), (5,6)
(3,2), (3,4), (3.6); (6,n, (6,3)" (6,5)
i.e., 18 cases in all.
18 I
Hence P(C) = 36 = 2"
\
Cases favourable to the events An B, A (') C, B (') C and A (') B (') C are
given below:
Event Fav,ql,Uable cases
AnB (1,1), (i l 3)~ (1, 5), (3,. '1), (3, 3), (3, 5), (5, 1) (5, 3)
(5,5), i.e., 9 in all.
A(')C (1,2), (1,4). (1.6), (3.2), (3, 4), (3, 6), (5, 2), (5, 4)
.(5,6). i.e., 9 in all.
B(')C (2, 1), (4,1), (6, 1) (2, 3), (4, 3), (6, 3), (2, 5), (4. 5).
(6. 5), i.e., 9 in alf
AnB(')C Nil, because AnB· implies that sum of points on two dice is
even and hence (AnB )(')C ell =
. 9 1
P(A (') B) = -36 = -4 = P(A).P(B)
9 1
P(A (') C) = -
36
= -
4
= P(A) P(C)
P(B.(') C) = 369 = 41 =P(B) P( C)
and P(A (') B (') C) = P(eII) = 0." P(A) P(B) P(C) _
Hence the events A, B and C are pairwise independent but not mutually
inde~ndent
•
Example 4·52. Let A.,Az, .... A. be independent events and P (At) =Pl.
Further, let P be the probabiJiJy thoi fWne of the events occurs; then sho'(fl thol
p ~ e - tPI [Agra Univ. M.Sc., 1987]
Visit : www.EasyEngineering.net
Visit : www.EasyEngineering.net
SOlutfon. We have
p = p ( AI 0 A2 n ... n A~)
I
II II II
and 0 ~ Pi ~ 1 ]
II
~ p ~ exp [- 1: pd,
i= 1
as desired.
Remark. We have
I-x ~ e- 1t for 0 ~ x ~ 1 ... (*)
Proof. The inequality (*) is obvious for x =0 and x = 1. Consider 0 < x < 1.
Then ~. .
1
log (I-xf = -102 (I-x)• <-
X2 x) X4 ]
.. [ x+2'+'3+'4+ ... ,
the expansion being valid since 0 < x < 1 . Further since x > 0, we get from (* *)
log(l-xr 1 > x
-log (l - x) > x
log (1 - x) < - x
~
as desired.
Example 4·53. In thefollowing Fig.(a) and (b) assume that the probability oJ
a relay being closed is P akd that. relay is open or closed independently of any
other. In each case find the probability that current flows from L to R.
~t~ ~2~R
It""
V' T·",'s FI9(&)
. H 6 F"U)
Solution. Let Ai denote the event that the relay =1,2, .•.,6) is closed. i, ( i
Let E be the event that current flows from L to R.
In Fig. (a) the current willflow from L to R if at least one of the circuitsirom
L to R is closed. Thus for the current to flow from L to R we have the fo119wing
favourable cases:
Visit : www.EasyEngineering.net
Visit : www.EasyEngineering.net
Visit : www.EasyEngineering.net
Visit : www.EasyEngineering.net
letter goes to the kth envelope but (n - 1) letters can go to the remaining (n - 1)
envelopes in (n - 1) ! ways.
Hence P (Ak) = (n - I)! = .!..
n! n
where P \AI:) denotes the probability of the kth match. It is interesting to see that
P (AI:) does not depend on k.
Example 4·54. (a) 'n° different objects 1.2 •...• n are distributed at random
in n places marked 1.2 •...• n. Find the probability that none ofthe objects occupies
the place corresponding to its number. [Calcutta Univ. B.A.(Stat.Hons.)1986;
Delhi Univ. B.sc.(MathS Hons.), 1990; B.Sc.(Stat.Hons.) 1988J
(b) If n letters are randomly placed in correctly addressed envelope~,prove
that the probability that exac~/y r leters are placed in correct envelopes is given by
1 II-! I: 1
-; 1: (-1) -k'; r=I.2 •....•. n
r. k=O •
Visit : www.EasyEngineering.net
Visit : www.EasyEngineering.net
.+ (_1)".,.1 '1 ]
n(n - 1) .. .3 .2. I
= 1- [1- 2\ + 3\ - ... + (- 1)~ -I n\]
1 1 1 ~ 1
=:=-2'--3'+-4,-···+(-1)
. .. -,
n.
n (-It
=k=O
I k'•
Remarlt For large n,
1 1 1
p= 1-1 +21-31+41- ...
= e- I = 0·36787
Hence the probability of at least one match is
1 1 (- 1)~
I-p=1-- 2.'+-3'-···+
. n.,
= 1-!, (foclarge n)
/ e
(b) [Probability or exactly' r matches {r ~ (n - 2) }] Let Aj , (i = 1,2, ... , n)
denote the event that ith letter goes to the correct env~lope. TIlen the l?robability
that none of the n letters goes to the correct ,envelope is
n
pal f"'lA2 f"'l ... f"'l AJ = E (- Itlk! ...(**)[(c! part (a)]
k=O
The probability that each of the 'r' letters is in the r~ght envelope is
n (n - 1) (n _ i) ... (n _ r + 1) , and the probability that none of the remaining
(n - r) letters goes in the correct envelope is obtained by replacing n by (n - r) in
n-r (_ It
(**) and is thus given by k:O k! . Hence by compound probability theorem,
the probability that out of n letters exactly r. letters go to correct envelopes, (in a
specified order), is
1 n-r(l~
E ~; r~n-2.
n(n-l)(n-2) ... (n-r+ 1) k=O k.
Since r letters can go to n envelopes in ~C, mutually exclusive ways, the
required probability of exactly r lettets going to correct envelopes, (in any Ofder,
whatsoever), is given by
Visit : www.EasyEngineering.net
Visit : www.EasyEngineering.net
Visit : www.EasyEngineering.net
Visit : www.EasyEngineering.net
1 a
Pk-Z = a+'b + 1 Pk-3
. + a+ b + 1 ,.. (3)
1 a
PZ = a + b + 1 PI +a +b+ 1 •.. (k - 1)
But p: = Probability of drawing a white ball from the first urn =~b
a+
.
equation by ( a + b + 1
1 ;-Z and adding, we get
Pi = (a +! + 1 fl PI ~
+ a +: + 1 [ 1 + a +! + 1 + (a + + 1)2 + ...
. ( 1
+ a+b+ 1
;-Z]
_ ( 1 ;-1 x __a_+ a [
(
l-la+b+ 1
I J
1- 1
1
- a +b + 1 (a + b) a + b + 1 ( 1 _,' 1 )
a+b+l
a
= li+b
( 1
a+b+l
;-1 a [
+ a+b 1- a+b+l
( 1 ;-1]
= a:b[(a+!+
a
Ifl +{ 1-(a+!+ Ifl}]
= - b ' (k=I,2, ... ,n)
a+
Since the probability of drawing a white ball from the kth urn is independent
of k, we-:have
a
p,,=--.
a+b
Example 4·56. (i) Let the probability p" that afamily has exactly n children
be a pit when n ~ 1 and po = 1 - a p (1 + P + pZ + .. :). Suppose that all sex distribu-
tions of n children have the same probability. Show that for k ~ 1, the probability
that afamily contains exactly k boY$ is 2 a .l/(2 - pr l •
Oi) Given that afamily includes alleast one bUy, 'show that the probability
that there are two or more boys is p/(2 - p).
Visit : www.EasyEngineering.net
Visit : www.EasyEngineering.net
and
·[Putj-,k=r.]
Hence
I -
at
(b) Let B denote the event that a family includes ieast one'boy and C denote
the eyent that a family has two or more boys. Then '
Visit : www.EasyEngineering.net
Visit : www.EasyEngineering.net
-i
- k=2
2al
(2:',:pt+ 1
- 2a
- 2-p
i
k=2'
.(~J
2-p,
2a . fp/(2 - p)]l ap2
= '2'-= p • 1 -
fp/(2 - p)] =. (2 - p)2 (1- p)'
Since C cB andB ('\ C= C,P (B ('\ C) =P (C) ~ P (B)P (C IB) =P (C)
Therefore,
P(C) ap2 (1-p)(2-p) p
P(CIB)=--= x -
P(B) (2_p)2(1_p) ap 2-p
Example 4·57. 'A slip of paper is given to person,A who marks it either with
a plus sign or a minus sign,' the probability ofhis writing a plus sign is 113. A passes
the slip to B, wlu? mqy either leave it alone or change the sign before passing it to
C. Next C passes the slip to D after perhaps changing the sign. Finally D passes it
to a ~eferee after perhaps changin"g the ~ign. The ref~ree sees a plus sign on the
slip. It is known that B, C and D each change the sign with probability 213. Find
the probability that A originally wrote a plus.
Sol~tion. Let us define ~e following events.: •
EI : A wrote a plus sign; £2: A wrote a minus sign
E : The referee observes a plus sign on the slip.
We are given: P (EI ) = 113", P (E2 ) = 1 - 113 = 2/3
We ~ant P (EI I E), which'by Bayes'rule is given,by:
P (EI ) P (E 1EI ) . "
P (Ell E) = P (EI) P (E 1EI~ + 'P (E2) P (~ 1E2) ... (i)
P (E 1EI ) =P [Referee observes the plus sign given that 'A' wrote
the plus sign an the slip]
= P [(Plus sign was not changed at all) v (plus sign was
.~hanged exacOy twice'jn,passing from 'A' to referee
through B, C and D)]
=P (£3 v £.), (say).
= P ~(~3) + f (~4), •.. (ii)
Visit : www.EasyEngineering.net
Visit : www.EasyEngineering.net
Let AI. Ai and A3 respcctivelydenote the events that 8, C and D change the
sign on the slip. Then WI# are given
P (AI) == P (AI) == P (A3):: 2/3 ; P <AI) == P (A2) =P <A3) = 113
We have
P (E3) =f (AI n A2 n A3) =P (AI) P (A2) P (A3) =(l/W =1127
P (E.) = P [(AI A2 A3) V (AI A2 A3) 1..,;1 (AI Al A3)] ,
=P (AI A2A3) + P (AI A2A3) + f (JI A2A3)
= P (AI) P (A2) P (A3) + P (AI) P (A2) P (A}) + P (AI) P (A 2) P (A3)
2212121224
. ==3·3·3 + 3·3·3 +. 3·3·3=9'
Substituting in (it) we get
'I 4 13
P (E lEI) =-+- = - ...(iii)
, 27 1) 27
Similarly,
P (E t£2) =P [Referee observes the plus sign given that 'A' wrote minus
sign on the slip]
=P [(Minus sign was chang~d exac~ly.once)
v (Minus sign was changed thrice)]
::; P (Es V E6), (say),
= P (Es) + P (£6) ,..(iv)
P (Tis) =' P [(AI A2 A3) V (AI A2 A3) V (AI A2 A3)]
= P (AI) p. (A2) P <A3) + P (AI) P (A2) P (A3) + P (AI) P'(A2)" P (A3)
2111-'211122
=3·3'"} + 3·3·3 + 3·3·3=9
2 2 2 8
P (E6) = P (AI A2A3) = P (AI) P (A 2) P (tt3) = 3·3·3 = 27
Substituting in (iv) we get:
2 8 14
P (E lEI) == "9 + 21 = 27· .. ,(v)
Example 4·58. Three urns of the >same appearance have the foliowing
proportion of balls.
-First urn 2 black' 1 white
Second Urn 1 black 2 white
Third urn 2 black 2 whit~
Visit : www.EasyEngineering.net
Visit : www.EasyEngineering.net
One ofthe uens is,selected and one Mil isdra'wn.lt turns out to be white. What
is the probability of drawing a white ball again, the first one not having oeen
returned?
Solution. Let us define the events:
Er= The'event of selection ofith' urn, (i = I ,2,~)
and A::: The event of dr;lwing a ~hite ball.
Then
P (E I ) = P (£z) = P (E3) = 1/3
and P (A rElj = 1/3, P (A I"Ez) = 2/3 ~nd P (A I E3) == 1/2
Let C denote the future event of drawing another white ball from the urns.
Then
P(CIElIlA) = O,P(CIEzIlA) = 1,1.andP(CIE3IlA) = h
3
. t P(Ei)P(AIEj)P(CIEiIlA)
P (C I A) = ...:;-=..:..1---=3:--------
t P (Ei) P (~ I f:"i)
;= 1
1 1 0 1 2 1 1 1 1
3'"3· +3·3'2+3'2'3 I
= 1 1 1 2 1 1 - 3
3 . 3 + 3 . 3 + "3 ;j
MISCELL.A.NEOUS EXERCISE ON CHAPTER IV
1. Probabilities of occurrence Of n independent events EI • Ez•... , E" are p.,
pz, ...• p .. respectively. Find tl)e probabili.ty of occurrence of the compound event in
which E •• Ez••••, E, occur r
ana E,. I. E,. z••••• E,. do not Occur.
r /I
Visit : www.EasyEngineering.net
Visit : www.EasyEngineering.net
Usc the abo\:'.,e relation tQ compute the probabil ity .that in si~ toss~s of a fair die, no
"aces are obtained". Compare this wi,th the pppcr bound given above. Show that if
each Pi is small tom pared. with n, the upper ~und is a gQOd approxim~tion.
-5. A and B 'play a t;natc~, the winner being the one who first wins two games
in succession, no games being drawn. Their re~pective chances of winn~ng a
particular game are' 'p : q. Find
" (i) A's initial chance of winning.
(ii) A's chance of winning after having won the first game.
6. A carpenter has a tool chest with two compartments, each one having a
locJ<,. He has two keys for each lock, and he keeps all four keys in the 'same ring.
His habitual procedure in opening Ii compartment ~s to select a key' at random' and·
try it. If it fails, ~e selects one of the remaining three and tries it and so on. Show
that the probability that he succeeds on the first, second and third try is 112,1/3, t:/6
respectively. (Lucknow Univ. B.Sc., 1990)
7. Three players A, Band C agree to playa series of gap1~ qbserving the
following rules: two players participate in each game, while third is idle, and the
game is to be won by one of them. The lose~ in each game quits and his place in
the next game is taken by the player who was idle. The player who succeeds in
winnin~ over both of his opponents without interruption wins the whole series of
games. •
Supposing the probabilIty for each player to win a single game is 112, and that
the first game is played by A andB, find the probability for A, B and C respectively
to win the whole series if tile numbef of games is unlimited.
Ans. 5/14,5/14,2(1 ,
8. In·a certain group of mathematicians; 60 per cent have insufficient back-
,gr9lPld of modem Algebra, 50 per cen,t have inadequate knowledge pf Mathemati-
cal Statist,ics and 80 per cent are in either o~e or both of the two categories. What
is the percentage of ~Qple who know Mathematical Statistics among those wh'o
have a sufficient background of Modem Algebra? (ADS. 0,50)
9. (0) If A has (n + I) and B has n fair coins, which they flip, show that
the probability that A geL<; more heads than B is .~.
rb) A stli<fent-'is given a column of IOdates and column'of 10 events·and is
asked to match the correct date to each evenc He is not allowed to use ariy ite'm
more than once. Consider the case where'the student knows. how to match four of
the items but he is very doubtful of the remaining six. He.decides to·match these
at random. Find the probabilities that.he will correctly match (i) all the items,
(ii) at·least seven of the items, 'and (iii) at least five.
1 10 I
Ans. (a)6!' (b)6T' (c) 1- 6!
10. An astrologer claims that he can predict before birth the sex of a baby just
to be born. Suppose that the astrologer has no 'real power but he tosses a coin just
Visit : www.EasyEngineering.net
Visit : www.EasyEngineering.net
once before every birth and if the head turns up he predicts a boy for that birth and
if the'tail turns up he predicts a girl. Let p be the probabilily of the event that'at a
certain birtti a male child is born, and p' the pro15ability of a head turning up· in a
single toss with astrologer's coin. Find the probability of a correct 'prediction and
that of at least one correct prediction' in' n predictions.
11. From a pack of 52 cards an even numbel: of cards is drawn. Show that the
probability of half of these cards being red is·
.[52 !/(26 !)l_ I] I (2s1 - I)
12. A sportsman's chance o( shooting an animal at a dista~ce r (> a) i~
al/rl.. He fires when r =2a, and if he misses he reloads and .f~res when
r = 30,40 •.. Jf he misses at distance na, the anitpal escapes. Find. the odds ag~inst
the sportsman.
Ans.n+I'n-1
Hint. P [Sportsman shoots ilt a distance' ia] = all =~
(io) i
~ P [Sportsman misses the shot at a distance ia] = 1 - .\
1
=~ (i-: i) ~
i=2 1 i=2
(i~ 1
I
)=!!±!
2n
Requn:edratio= n2+nl : ( 1- n2+nl )= (n+ l),: (n-l)
13. (0) Pataudi, the captain of the Indian team, is repoi1ed to have observed
the rule of calling 'heads' every time ,the to~ was made during the five'matches of
the Test series with the Austral~ team. What is the probability of his wirming the
toss in all the five matches?
Ans. (l/2)s .
How will the probability be affected 'if '.
(i) he had made a rule of tossing a coin privately to dec.K\e whether to call
"hea~lt or "tails" on each,occasion.
(a) the factors deteqnining his choice were not pre<tetermined·but he called
:ouUihatever occWYoo to him on the spqr of the moment?
(b) A lot contains SO defective and 50 non-defective bulbs. Two bulbs are
drawn ~t random ~e at a- time. ·with replacement 1)e events A, B'. r. are
defillfAas
A = (The first bulb is defective)
B = (The second bulb is non-defective)
C= {The two.bulbs are bodt 4efective or both non-defectiv~}
Visit : www.EasyEngineering.net
Visit : www.EasyEngineering.net
Determine whether
(i)A. B, C are p;urwise independent,
(ii)A. B, C are independent
14. A, B and G are three urns which contain 2 white, 1- black, 3 white. 2 black
and 2 white and 2 black balls. respectively. One ball is drawn from urn A and put
into the urn B; then a ball is drawn from urn B and put into the urn C. Then a ball
is drawn from urn C. Find the probability that the ball drawn is while.
Ans. 4/15. .
15. An urn contains a white and b black balls and a ~ries of drawings of one
ball at a time is made. the bail remove(J being retrurned to the urn 'immediately after
the next drawing is made. If p" denotes the probability that the nth baH drawn is
blade. show" that
p~ :-(b - P~-t) I (a + b- '1).
Hence nnd Pit •
16. A person is to be tested to see whettter he can differentiate between the
taste of two brands of cigarettes. If he cannot differentiate. it is a~sumed that the
probabili'ty is one-half that he will identify a cigarette correctly: Under which'of
the following two procedures is ther:e less cpance ~hat he will make all correct
,identifications when he actually cannot differentiate between, the two br,ands?
(i) The subject i~ giv~n fo!JI' p~rs each containing bo"th"brands.of cigarettes
(this is known to the subject). Jte !.11~~t identify for each pair which cigarette
represents each.brand.
'(ii) The subject is given eight cigarettes and is told that the first four are of one
brand and the last four of the other brand.
. How do you explain the difference in results d~spite the fact ~at eight
cigarettes ~e tested in each case?
Ans. OJ 1/16 (ii) l/2
17. (Sampling with replacement). A sample of size r is takep (rpm a
popu\atipn of n people. Find the probability.'Vr that N given people will be inc'Iu~.ed
in. the, sample. . -
Visit : www.EasyEngineering.net
Visit : www.EasyEngineering.net
19. '.In a certain book of N pages. no page contains more than four errors, nl
of them contain one error, nl contain two errors, n) contain three error:; and n.
contaill four ~rror~. 'FwQ copies of the book are opened at any' two g~ven ,pages,
Show the pr9bability that the number of errQ{S in these two pages ~hall not-exceed
.five is
1 -= ~ (n31 + nl + 2nl n. + 2n3 n.)
N
Hint. Let Ei I : the event that a ,page of first book contains (errors ..
and Ei II : the event that a page of second book contains i errors,
p ~o. of errors in the two pages shall. not exceed 5)
;:;:.1 - P [Gl I P4 II + E3 I E4 II +. E. I E4 Ii
+ E3 I E) II + E4 I E3 II + E4 :~ El II 1.
20. (a) Of three independent even~. the chance that the ftrst only should
, happens is a, the chance of the, second only is b and the chance of tbe third only
-is c. Show that the independent chances of the three events are reSpeCtively ~I
l b
_0_ _ _ _ c_ '
. ~o.+r·'b+x· c.fx I
.....
flrnt P (EI ("\ £1'("\ '£3) =P (E 1) U - p. (Ei)] [1 ...,. P (£3)] =a
P (£1 ("\'£1'("\ Ej) = [1- P (E1)] 'P (El) [1- P (E3)] = Ii ...( )
P
-p (EI ("\ E~ ("\ £3) = ['i - P (£1)] {l- (El)] P (E~) c = ...( )
Multiplying (.), (..) and (~ ..), we get
. p (£1) P (E1)P (E~) x' l = abc,
w~re.x-= [1 - P (E 1)] [1 :- P. (E1)] [1 -.P (E3)] ,,
Multiplying (.) by [1 - P (EI~].-we get
r(E 1) =~
o+x
,and so on.
(b) Of three independent events, the probability that the ftrst' only should
happens is 1/4, the probability that the 'Second only should happen is 1/8, and the
probability that the third only should happen is 1/12. Obtain the unConditional
~bilit1es of the three events.
Ans. 112, 113. 1/4.
(c) A total of n shells are fared at a target The probability of the ith shell ,hitting
tIie target is Pi; (= I, 2, 3, .." II! I\ss~ming that the II firings are n mutually
independent events, find theJKobability that.at least two shells out of Whit the
target. [Calcutta Univ. B.sc.(Maths Hon~), 1988]
(d) An urn cOntains M balls numbered, 1 to M, where the ftrst K balls are
defective and the remaining M .... K· are non4efective. A sample, ~f n balls is
~wn f.roin the~. Let At be the event Jhat the sample of'II balls contains exactly
k defeCtives. ruUt p(At) when the sample is drawn (i) with replacement and.
(a) withoot replacemenL [Delhi Univ. B.Sc. (Maths HonS.), 1989]
Visit : www.EasyEngineering.net
Visit : www.EasyEngineering.net
21. For three independent events A; 8 and C, the probability for A to oc~ur·i!\
a, the probability that,( 8 and C will not occur is b, and the probability that at least
one of the three events will not occur is c. IfP denotes the probability that C occurs
but neither A nor 8 occurs, prove that p satisfies the quadratic equation
ap~+ [ab- (1- a) (a+c - I)] p=+-b (I-a) (1- c),=O
. (l-a(+ab
and hence deduce that c > (I _ a) >
Further.show/that tlte probability of occurrence of·C is p/(P + b); and that of
8's happening is (I -.c) (p + b)/ap.
Hint. Let P (A) =x, P (8) = y and P (C)~ ~
Then x=a, (1-:t)(1-y)(l-z).=b, l:-xyz=c
and \ - p=z(I-.x)(I;-y)
Elimination of x, 'y and z' gives quadratic equation in p.
22. (a) The chance' of success in each trial is p. If Pi is the probability ·that
there are even number of successes in k trial,s, prove that
Pi =P + Pi-I (1 - 2p)
Deduce that Pi = i:[1 + (1 - ZPt]
(b) If a day is dry, the conditional prob~bility that the following day wlil also
be dry is p; if.a day is wet, the conditioruil probability that 'the foUowing day will
be dry is p~. it u.. is the probability that the nth day will be dry, prove that
u,.-(P-P')u..-I-P'=O; 'n~2
If the fust day is dry, p = 3/4 and p' ='114, fmd u,. •
23. There are n similar biased dice suct..that ~e prol:>ability of obtaining a 6
with each one of them is the same and equal to p. If all the dice are rolled once,
show that p", the probability that an odd number of 6's is obtained satisfies the
difference equation •
p,. + (2p - 1)·p"_I~= P
and hence deriye'an explicit expresSioii for p".
'ADS. p,,=![l".f(1-2p)"]
1 r
24. SUIJIX)se that each day tl)~ weath.er,can be uniquely classified as '(me' or
'bad'. S~ppose further that the probability of haVing fme wcilther on ~~ Jas,t day
of fl ~~ year i$ Po and. we have the prQbability p'that the weather on an arbitrary
day will be ,ofthe same lUnd as on the preceding day. Let the probability of having
fme w~ther on the nth day of the following year be PII' Show that .
P:=(2p-I)P,,-I+(I-p) ~- •
~ucethat
. ,( I)' +'21
P,=;(2p-I) Po -'2
25. A closet contains n Pairs .of ~~s. If 2r shoes are chosen at random
(with 2r < n ), wh~t is the probability. '~' there -will be (i)'oo complete pair,
Visit : www.EasyEngineering.net
Visit : www.EasyEngineering.net
(ii) exactly one complete pair, (iii) exactly two complete pairs among them?
Hint. (i) p(n~ complete pair)= ( ;r ) 2'Jr +, ( ~ )
.26. ShQw.that the probability of getting no right pair olit of n, when the left
foot shoes are paired randomly with the rigth foot shoes; is the sum of the frrst
(n + I) tenns in the expansion of e- I •
27. (a) In a town consisting of (n + I) inhabitants, a person narrates a rumour
to a second person, who in turn narrates it to a third person, and so on. At each step
the recipient of the rumour is chosen at raJ)dom from the n available persons,
excluding .the narrator himSelf. Find the probability. that the rumour will be told r
times without:
(i) returning to the originator,
(ii) being narrated to any person more than once.
( b) Do the above problem when, at each step the rumour is told by 09.e p€(rson
to a gathering of N randomly cfi9sen people.
( )( .)n(n-l),-I --
A ns.a, (I-!J-I. (. ).
,Il
n(n-I)(n-2) ...(n-r+ l )
- -.-
n' n nr
. )t~)
(lim
28. What is the probability that (i) the birthdays of twelve people will fall
in twelve different calendar months (assume equal probabilities for the twelve
months) and (ii) the birthdays of six people will fall in exactly two calendar months?
Hint. (i) The birthday of the fllSt person, for instance: can fall in .12 different
ways and so for the second, and so on.
:. The total number of cases = li2.
Now there are 12 months in which the birthday ,of one persOn can fall and 11
months in which the birthday of the second pe~n can fall and 10' months f6r
another third person, and so on.
:; The total number of favourable cases::; 12.11.10.. .3.2.1
Hence the required probability = B.!
• .' 1212
The total number of ways in which tl'.e birthdays of 6 persons can (all in
(ii)
any of the month = It.
, (.t2] (2
6- 2)
The·required probability = ~.' 126
Visit : www.EasyEngineering.net
Visit : www.EasyEngineering.net
29. An elevator starts with 7 passengers and' stops at 10 floors. What is the
probability p that no two passengers leave at the same floor? ,
[Delhi Univ. M.e.A., 1988]
30. A bridge player knows that his two opponents have exactly f1ve hearts
between two of them. Each opponent has thirteen cards. What is the probability
that there is three-two split on the hearts (that is one player has three hearts and the
other two)? [Delhi Univ. B.Sc.(Maths Hons.), 1988]
31. An urn contail)s Z white and 2 black- balls. A ball is drawn at random. If
it is whitt. it is riot replaced into the urn. Otherwise it is replaced along with another
ball of the same colour. The process is repeated. Find the probability that'the third
ball drawn'i,s black. [Burdwan Univ. B~Sc. (HODS.), 1990]
23
Ans. 30
32. There is a series of n urns. In the itll: urn there are i 'white and (n -I)
black balls. i == '1. 2. 3..... k. One urn is chosen at random and 2 balls are drawn
from it. Both turn out to be white. What is the probability that the jth urn was
chosen. where j is a particular number berween 3 and n.
Hint. Let Ej denote the event of selection of jth urn. j =3. 4..... n and A
denote the event of drawing of 2 white balls. then
P(AIE-)=(i)(cl).
J II 11-1
P(E-)=!
J II'
P(A)= •
1=
i 1"1 (i..)(i=.!)
II 11-1
P(EjIA)= _
!( i )( cl )
II II 11-1
i~.(~)(*)(!~~)
_33. There are (N + I) identical urns marked O. I. 2..... N each of which
contains N white and red balls: The kth urn contains k red and N - k white balls.
(k =0; I. 2•... N). An-urn is chosen ~t random apd n ~d9m drawmgs of a ball are
made f~m it, the ball drawn being replaced after each draw. H the balls drawn are
all red. show that the probability that the next drawing will alsQ yield a r~ ball is
approximately (n + I) (~ + 2) when N is large.
34. A printing machine can print n letters. say al. al..... a. . It is operated
by electrical impulses. each 'etter being prodiJced by a different impulse. Assume
that p is the constant probability of .printing the correct letter and :the impulses are
independent. One of the n-impulses. chosen at random. was fed intO-the machine
twice a..1l<1 both times the letter ai was printed. Compute the· probability that the
impulse chosen was meannoprint al. [Delhi Univ. M,sc.(Stat.), 1981]
Ans. (n_l)pl/(npl_2p+ I) , ,
35. Two playC'lS A and B- agree to conlest a match consisting-of a'Series of
games. the_ match to- be won by the player who rust wins three games. with the
provision that if the players win two gam~ each. the-match is to continue until it
Visit : www.EasyEngineering.net
Visit : www.EasyEngineering.net
is won by one player winning two games more than his opponent. The probabililty
of A winning any given g~e is p,"and the games cannot be drawn .
.(i) Prove thatf(p). the initial probability of A winning the match is given by:
f(P) =p3 (4 - 5p + 2l)(1-7/H.'J,p2)
,0;) Show that the equation f (p,).= p has five r~ roots, Qf whi~tt.' t~r~e are
adrpissible values of p. Find these three· roots and explain their significance,
[Civil Services (Mai.n), 1986]
36. Two. players A and B start playins. a series of games with J.?s. a ;md b
respective~y. The stake is Re. I on a game and no game can be drawn. If the
probability pf A 'Yinning any game is a <;Ot:lstaQt p, find the initi~ proQapility of
.his exhausting the funds of B or his own. Also show that if the resources of B
ru:e ut:lHmited then
(i) A is certain to be ruined if p = l,1 , and
(ii) A has an even chance of escaping ruin if p =tl·/O + tl.)~
Hint. Let u" be the probability of A's final win when he has Rs~1,I.
Thep u,. =pu,,,+1 +(1- p)Y,,-.i where Un =O· and· u,. H'= I
u,.+1-u,.=r,P
1.=.£)(U,,-u,._I)
.
Hence u,.,+.1 - u,. =( I; P J Ult by repeated applicatipn,
so that
Visit : www.EasyEngineering.net
Visit : www.EasyEngineering.net
Also U]I= ( 43 -3'1) U__ I +41 U_-z => u" +3'1 Un-I =43( U_-I +3'1 Un -2 )
This equation can be solved as a homogeneous difference equation of second
order with the initial conditions
1 1 5 5
Uo =:3' UI = 3' '}2 = 36
38. The following weather forecasting is used by an amateur forecaster. Each
day is classified as 'dry' or 'wet' and.the probability that any given day is same as
the prec~ing one is assumed to beaconstantp, '(0 <p < 1). Based on past records,
it is supposed that January 1 has a. probability ~. of being dry. Letting
~_ = Probability that nth day'of the year is dry, obupn an ~~pressipn for ~n in
terms of ~ and p, Also evaluate lim, ~_.
Hint. ~_ = p.~n-I + (I-p)(1-~ __ IJ
=> ~_ = (2p- D ~_-I + (I-p) ; n = 2,3,4, ...
l'\ns. ~. = (2p - 1r -I, (~ - Ill) + Ill; lim ~_ =.III
11-+00
39. Two urns cQntain respectively 'a white and b black' 'and 'b .white and a
black' balls,. A series of drawings is made according to the following:rules:
(i) Each time only one~ball is drawn and imm,ediately retufued to the same urn
itcame from.
(ii) If the ball drawn is white, the next drawing is' made from the first urn.
(iii) If it is black, the next drawing is made from the second urn.
(iv) The first ball drawn comes from the first-urn.
What is the probability that nth ball drawn will be white?
Hint. p, = P [Drawing a white ball at the rth drawJ.
, a b· )
p, ,= ~bP.'-1 '+ --b'( I-p,,_1
a+" a+ "
a-'b b "
=> ""i+iJ. P,-I + ~ + b
p, =
Visit : www.EasyEngineering.net
Visit : www.EasyEngineering.net
Visit : www.EasyEngineering.net
Visit : www.EasyEngineering.net
(iii) A coin is tossed three .times in succession,. the number of sample points
in sample space is
(a) 6 (b) 8 (c) 3
(iv) In the simultaneous tossing of two perfect coins, the probability- of
having at least one head is
(a) !2 (b) !4 (c)' 14 (d) 1
(v) In the simultaneous tossing of two perfect dice, the p'robabili~y' of
obtaining 4 as the sum of the resultant faces is .
4 1 3 2
(a) 12 (b) '12 (c) 12 (d) 12
(vi) A singlp.leu~r is selected at random from the word 'probability'. The
probability that it is a vowel is
3, 2 4
(a)1l (b)1l (c) Il (d) 0
(vii) An lD1l contains 9 balls, two of which are red, thre.e blue and four
blaCK. Three balls are drawn at random. The chance that they are of the. same
colour is
(a)'~ (b) ~ (c)' ~ (d) :7
(viii) A number is chosen ,at random among the first J20 natural numbers.
The probability of the number chosen being a Multiple of 5 or 15.is '
1 1 1
(a) 5 (b) '8 (c) 16
(ix) If A-and B are mutually exclusive' events, then
(a) P(AuB)=P(A).P(B)
(b) P(AuB)=P(A)+P(B)" (c) P(AuB)=O.
(x) If A and iJ are tWQ independent events,· ,th~ probabili~y~that both A
i
and B occur is and the probability that ~either of them occUrs is.~. The prob-
ability of the occurrence of Ai,s:
1 1 1
(a) 2' ,(b) 3I (d) :s'
VI. Fill in the blanks;
·(i) Two events are $lid·to be equally likely if .... ..
(ii) A set of even~ is said to be independent if ..... .
(iii) If P(A) .. P(B). P(C) =f(A nB nC). then the, events A, B,.C are ..... .
(iv) Two events·A 'and B are mutually exclusive if P (1\ n,B) ='" and are
independent if P (A n B) ='" .
(v) The probability of getting a multiple of 2 in a throw of a dice is 1/2 and
of getting a multiple of 3 is 1{3. Hence probability of getti,ng a multiple of 2 or 3
is ......
(vi) Let A and B be independent events and suppose the evtrpt C has prob-
ability 0 or 1. Then A, Band Care ...... events. ..
(vii) If A, B, C are papwise independent and A is independent of B u C.
then A, B, C are ...... independent.
Visit : www.EasyEngineering.net
Visit : www.EasyEngineering.net
(viii) A man has tossed 2 fairdiee. The conditional probability that he has
tossed two sixes, given tha~ he has tossed at least one six is ..... .
(ix) Let A and B be two events such that P (A) =·0·3 and P (A v B) = 0·8.
If A and; B are independent events then P (B) ='" '
VII. Each of following statements is either true or false. If it is true prove-it,
otherwise, give a counter example to show that it is false.
OJ The probability of occurrence of at least one Qf two events ,is the sum
of the probability of each of the two events.
(ii) Mutually exclusive events are independent.
(iii) For any two events A and B, P (A ("\ B) cannot be less than either P (A)
or P (B).
(iv) The conditional probability of A given B is always. grea~r than P (A).
(v) If the occurrence of an even\A implies the occurr~nce of another event
B then P (A) cannot exceed P (B).
(vi) For any two events A andB, P(AvB) cannot-be greater theneither
P (A) or P (B).
(vii) Mutually exclusive events are not independent.
(viii) Pairwise independence·does not necessarily imply mutual independ-
ence.
, (ix) Let A and B ~ events neither of Whic~. hils prObability zero. Then if A
and iJ are disjoint, A and B are independent. "
(x) The probability of any event is always a proper fraction.
(xi) If 0 < P (B) < I So that P (t\ l.fl) and ,P (A Iii) ar~ bQth defined, then
=
P (A) P (B) P (A IB) + P (Ii) P (A Iii).
(xii) For.two events A and B if
P (A)=P'(A IB) =·1/4 andP (A I B)' = 112;, then
(a) A and B are muwally exclusive.
-(b) A and B are independent.
(c) A is a sub-event of B.
=
(d) P (A IB) 3/4. l[Delhi Univ~ B.Sc.(Sta~ Hans.), 1991]
(xiii) Two eventS can be independent and mutually excliJSive simultaneously,
(xiv) Let A and B be even~, neither of which has p(Obal>ility zero. Prove or
disprove the following:
(a) If A llnd B are diSjoint, A ,and.fl are independent.
(b) If A and B are indepeodent A and B ,are disjoint
3
Visit : www.EasyEngineering.net
Visit : www.EasyEngineering.net
CHAPTER FIVE
"
Random Variables - Distribution Functions
5·1. Random Variable. Intuitively by a random variable (r.v) we mean a
real number X connected with the outcome of a random experiment E. For
example, if E consists of two tosses of a coin, we may consider the random varilble
which is the number of heads ( 0, 1 or 2). .
Outcome: HII liT Til IT
Value 0/ X : 2 I
,(01
1 o
Thus to each outcome 0> , there corresponds a real number X (0)). Since the
points of the sample space S correspond to outcomes, this means that a real number ,
which we denote by X (0)), is defined for each 0> E S. From this standpoint, we
define random variable to be a real function on S as follows:
.. Let S be the sample space associated with a given random experiment. A
real-valued/unction defined on S and taking values in R (- 00 ,00 ) is called a
olle-dimensional random variable. If the/unction values are ordered pairs o/real
numbers (i.e., vectors in two-space) the/unction is said to be a two- dimensional
random variable. More generally, an n-dimensional random variable is simply a
function whose domain is S and whose range is a collection 0/ n-tuples 0/ real
numbers (vectors in n- space)."
For a mathematical and rigorous definition of the random variable, let us
consider the probability space, the triplet (S, B, P), ~here S is the sample space,
viz., space of outcomes, B is the G-field of subsets in S, and P is a probability
function on B.
Def. A random variable (r.v.Y is a function X (0)) with domain S and range
(__ ,00) such that for every real number a, the event [00: X (00) S; a] E B.
Remarks: 1. The refinement above is the same as saying that the function
X (00) is measurable real function on (S, B).
2. We shall need·'to make probability statements about"a'random variable X
such as P {X S; a}. For the simple example given above we sbould write
p (X S; 1) =P {HH, liT, TH}.= 3/4. That i's, P(X S; a) is simply 'the probability
pfth~ set of outcomes 00 for which X (00) S; a or'
p (X S; a) =P { 00: X (oo)S; a)
Since Pis a measure on (S,B) i.e., P is defined on subsetsofB, theabovepro~bility
will be defined only if [ o>:X (OO)S;~) E B, which implies thatX(oo) is a measurable
function on (S,B).
3. One-dimensional random variables will be denoted by capit8I leuers.
X,y,z, ...etc. A typical outcome of the experiment (i.e., a typical clement of the"
sample space) will be denoted by 0> or e. Thus X (00) represents the real number
which the rand<,>m variable X associates wi~ the outcome 00. The values whict
X, y, Z, ... etc., can assume are denoted by lower case letters viz., x, y, z, .:. etc.
Visit : www.EasyEngineering.net
Visit : www.EasyEngineering.net
4. Notalions.-If X is a-real number, the set of all <0 in S s!lch that X( <0 ) =x is
denoted briefly by writing X =x. Thus
P (X =x) = p{<o: X (<0) =x.}
Similarly P (X ~ al= p{<o: X (<0) E l- 00, a]}
and P>[a < X ~ b) = P ( <0 : X ( (0) E (a,b] )
Analogous meanings are given to
P ( X = a or X= b ) = P { (X = a ) u ( X = b ) } ,
P ( X =a and X = b ) = P { ( X== a ) n (X = b )}, etc.
Illustrations: 1. If a coin is tossed. then
S = {<01, (Jh} where <01 = It, (Jh::; T
X(<o)=
_
{I, ~f <0 = N
0, If <0 == T
X (<oj is a Bernoulli random variable. Here X (<o).takes only two values. A random
variable which takes only a finite number of values is called single.
2. An experiment consists of rolling a die and reading the number of points
on the upturned face. The most natural random variable X to cunsider is
X(<o) = <0; <0 =1, 2, ... ,6
'. If we are interested in whether the number of point,s is even or odd, we consider
a random. variable Y defined as follows:
y ( <0 ) = {O,1, ~f <0 ~s even
if <0 IS odd
3. If a dart is thrown at a circular target. the sample space S is the set of all
points w <.'n the target. By imagining a coordinate system placed on the target with
the origin at the centre, we can assign various random variables to this experiment.
A natural one is the two dimensional random variable which ~signs to the point
<0, its rectangular coordinates (x,y). Another is that which assigns <0 its polar
coordinates (r, a ). A one dimensional random variable assigns to each <0 only one
of the coordlnatesxory (for cartesian system), rora (for polar system). Theevent
E, "that the dart will land in the first qUadrant" can be described by a. random
variable which a<;signs to each point'W its polar coordinate a so that X (<0) = a and
°
then E ={<o : ~ X (<0) ~ 1t12}. '
4. Ifa pair of fair dice is tossed then S= {1,2,3,4,5,6}x'{1,2,3,4,5,6} and
n (S) = 36. Let X be a random variable with image set '
XeS) =(I ,2,3,4,5,6)
P(X= 1)=P{I,I} = 1/36
P(X = 2) = P{(2,1),(2,2),(l,2)} =3/36
P(X = 3) =P{(3,1).(3,2),(3,3),(2,3).(l,3)} = 5/36
Visit : www.EasyEngineering.net
Visit : www.EasyEngineering.net
Visit : www.EasyEngineering.net